Hindi Insights IAS Prelims 2020 Subject Wise Test 4 @UPSC PDF

You might also like

You are on page 1of 67

https://telegram.

me/UPSC_PDF
Total Marks : 200
Online Prelims Full Length Test - 4
( InsightsIAS Mock Test Series for UPSC Preliminary Exam 2020 ) Mark Scored : 0

1 भारत म माइोफाइन स संथानों (MFI) के संदभ म िननिलिखत कथनों पर िवचार कीिजए :


1. MFI को गै र-लाभकारी सं था के प म पं जीकृत नहीं िकया जा सकता है ।
2. MFI ारा िदए गए ऋण मु य प से समाज के कमजोर वगो,ं िजसमे अिधकाँ श मिहलाओं की आवयकता पूरी करते ह ।
3. वतमान म MFI भारत म माइो ेिडट (सूम ऋण) का सबसे बड़े दाता है ।

नीचे िदए गए कू ट का उपयोग करके सही उर चु िनए :


A. केवल 1
B. केवल 2 और 3
C. केवल 2
D. 1, 2 और 3

Your Answer :
Correct Answer : C

Answer Justification :

ब को ारा िबना ेिडट-इितहास वाल को ऋण दे ने की अिनछा के पिरणामवप 90 के दशक म ग ामीण समु दाय को
सं गिठत करने व ऋण जोिखम उठाने की इछुक माइोफ़ाइन स सं थाएँ (MFI) अितव म आ।

माइोफाइन स े , िवशे ष प से वष 2000 के बाद से इसके कायाकप को दे खते हुए, PSB के यवसाियक माडल से िभन
माडल के उपयोग के तर पर सामिजक लय को ात करने का एक उम उदाहरण तु त करता है । अिधकां श
माइोफाइन स सं थान (MFI) ने अलाभाथी सं थाओं के प म अपनी शु आत की है । ‘बं धन ब क’ आरभ म एक MFI था
जो बाद म एक ब क म पिरवितत हो गया।

भारत म सोसाइटी तथा गै र-सरकारी सं गठन को िमलकर 223 MFI ह और उनम से 168 ‘सा-धन’, (सामु दाियक िवकास िव
सं थान का सं घ) के साथ पं जीकृत ह । माइोफाइन स इं टीटूशंस ने टवक (MFIN), एक उोग िनकाय के अं तगत
पं जीकृत 47 गै र-ब क िव कंपनी-माइो फाइन स सं थान (NBFC-MFI) ह , जो 90%पोट फोिलयो को कवर करते ह , िजनमे
से शीष 10 सं थान ारा इिवटी या ब क ऋण ात करना आसान होता है । छोटे िवीय सं थान हमे शा एक नु कसान म
रहते ह ।

सं युत राट ारा वष 2005 को माइोफाइन स वष की घोषणा ने गरीबी उमूलन म MFI की भूिमका पर काश डाला। कुछ
MFI ने खु द को ब क म भी तदील कर िलया है । 2016 तक, ऋण-ातकताओं म से 97 ितशत अनु सिू चत जाित /अनु सिू चत
जनजाित तथा अपसं यक समु दाय की मिहलाएं थीं। इससे यह पट होता है िक MFI ारा िदए गए ऋण मु य प से
समाज के कमजोर वगो,ं िजसमे अिधकाँ श मिहलाओं की आवयकता पूरी करते ह ।

जून तक, ब क 38,486 करोड़ पये के ऋण के साथ माइो ेिडट के सबसे बड़े दाता ह । यह MFIN के अनु सार, माइो-
ेिडट यवसाय म कुल िदए गए ऋण का 36% है । बं धन ब क, 21,400 करोड़ पये के ऋण पोट फोिलयो के साथ मु ख MFI
है ।

https://economictimes.indiatimes.com/industry/banking/finance/microfinance-institutions-are-strugg
ling-for-survival-heres-why/articleshow/60931020.cms?from=mdr

2 िलंगायतों ने िननिलिखत म िकसके िव तक िदए


1. सं कृत ु ित परं परा
2. जाित यवथा
3. मूित पूजा

https://telegram.me/UpscPdfDrive
https://telegram.me/UPSC_PDF
Total Marks : 200
Online Prelims Full Length Test - 4
( InsightsIAS Mock Test Series for UPSC Preliminary Exam 2020 ) Mark Scored : 0

4. भित सं त ारा मं िदर के अनु ठान की उपे ा

नीचे िदए गए कू ट का उपयोग करके सही उर चु िनए :।


A. केवल 2
B. केवल 2, 3 और 4
C. केवल 1, 2 और 3
D. उपरोत म से कोई नहीं

Your Answer :
Correct Answer : C

Answer Justification :

बारहवीं सदी के मय म कनाटक म बासवना एवं उनके अनु याियय, अलामभु तथा अका महादे वी आिद सं त ने वीरशै व
आं दोलन की शु आत की

सभी मनु य की समानता और जाितयवथा के बारे म बाणवादी िवचार के िव तथा मिहलाओं के उार के िलए दृढ़ता
से तक िदए।

वीरशै व ने िजसम जाित, धम या ीपु ष का भे दभाव न रहे । वह कमकांड सं बंधी आडं बर का िवरोधी था और मानिसक
पिवता एवं भित की सचाई पर बल दे ता था। वह मा एक ईवर की उपासना का समथक था और उसने पूजा तथा यान की
पित म सरलता लाने का यन िकया। जाित भे द की समाित तथा िय के उथान के कारण समाज म अद्भुत ां ित
उपन हो गई। ानयोग भितयोग तथा कमयोग - तीन वचनकार को माय ह िकंतु भित पर सबसे अिधक जोर िदया
जाता है ।

वे सभी कार के अनु ठान और मूित पूजा के भी िखलाफ थे ।

https://www.oxfordbibliographies.com/view/document/obo-9780195399318/obo-9780195399318-01
52.xml

3 िननिलिखत कथनों पर िवचार कीिजए :


1. "िवु त" और "जल िवु त पिरयोजनाओं से उपन िबजली" सं िवधान म राय सूची के अं तगत सूचीब ह ।
2. ‘जल’ रायसूची का िवषय है अतः राय जल िवु त पिरयोजनाओं का कायावयन राय नीितय से सं चािलत होता है ।

नीचे िदए गए कू ट का उपयोग करके सही उर चु िनए :


A. केवल 1
B. केवल 2
C. 1 और 2 दोन
D. उपरोत म से कोई नहीं

Your Answer :
Correct Answer : B

Answer Justification :

‘िवु त’ और ‘जल िवु त पिरयोजनाओं से उपन िबजली’ सं िवधान म समवती सूची के अं तगत सूचीब ह ।

https://telegram.me/UpscPdfDrive
https://telegram.me/UPSC_PDF
Total Marks : 200
Online Prelims Full Length Test - 4
( InsightsIAS Mock Test Series for UPSC Preliminary Exam 2020 ) Mark Scored : 0

‘जल’ रायसूची का िवषय है अतः राय जल िवु त पिरयोजनाओं का कायावयन राय नीितय से सं चािलत होता है ।
SHP पिरयोजनाएं तथा इसके आवं टन का िनणय राय सरकार ारा िलया जाता है । िनजी डे वलपस से िनिवदा / ताव /
बोिलयां राय सरकार ारा आमं ित की जाती ह ।

इन े  को िवकिसत करने तथा पिरयोजनाओं के आवं टन के िलए राय की अपनी नीितयां और ियाएं होती ह । सं बंिधत
राय सरकार ारा SHP पिरयोजनाओं के िलए तकनीकी-आिथक मं जरू ी (TEC) / अनु मोदन दान िकया जाता है ।

िबजली मं ालय, दे श म छोटे पनिबजली े  के िवकास के िलए नोडल मं ालय होने के नाते िविभन उपाय के मायम से
े  के िवकास को ोसािहत करने िलए यापक परे खा दान करता है । इनम SHP मता मूयांकन के िलए सहायता
आिद शािमल होते है , िजसम माइो िसिटं ग, िवकासशील परीण और मानकीकरण और िशण सु िवधाएं , सव ण और
जांच के िलए सहायता, िवतृ त पिरयोजना िरपोट (डीपीआर) तै यार करना, पिरयोजनाओं के िलए पूंजी सिसडी, जीणोार
और आधु िनकीकरण के िलए सहायता आिद दान की जाती है ।

https://mnre.gov.in/img/documents/uploads/743debe1e34e40a296ca036bc5656dc0.pdf

4 पे िनिसिलन के संले िषत पांतरण िननिलिखत से कौन से ह ?


1. एिरथोमाइिसन (Erythromycin)
2. एमोिसिसिलन (Amoxycillin)
3. लोरै मफेिनकॉल (Chloramphenicol)

नीचे िदए गए कू ट का उपयोग करके सही उर चु िनए :


A. केवल 1
B. केवल 2 और 3
C. केवल 2
D. 1, 2 और 3

Your Answer :
Correct Answer : C

Answer Justification :

कुछ एं टीबायोिटक से भािवत होने वाले बै टीिरया या अय सूमजीव की े णी को एं टीबायोिटक का िया पे ट म कहा
जाता है ।

जो एं टीबायोिटक मु य प से जम पॉिजिटव या जम- िनगे िटव बै टीिरया के िव भावी होते ह , वे एं टीबायोिटक के
सं कीण पे ट म ह । यिद ये िकसी एकल जीवाणु अथवा रोग के िव भावी होते ह , तो उह सीिमत पे ट म एं टीबायोिटक
दवाओं के प म सं दिभत िकया जाता है ।

पे िनिसिलन G एक सं कीण पे ट म है । एपीिसलीन और एमोिसिसिलन पे िनिसिलन के संले िषत पांतरण ह । इनम यापक
पे ट म होता ह । एं टीबायोिटस सूमजीव को या तो नट करते है अथवा रोगाणु ओं पर थै ितक (िनरोधामक) भाव
डालते है ।

5 ‘लू वे फर’ (Blue Wafer) तथा ‘लै क वे फर’ (Black Wafer) सौर फोटो-वोटाइक (PV) कोिशकाओं के बीच या अंतर
ह 
1. लू वे फर घरे लू प से उपािदत िकया जाता है जबिक लै क वे फर का आयात िकया जाता है ।
2. लै क वे फर सामायतः लू वे फर की तु लना म अिधक ऊजा मतावान होता है ।

https://telegram.me/UpscPdfDrive
https://telegram.me/UPSC_PDF
Total Marks : 200
Online Prelims Full Length Test - 4
( InsightsIAS Mock Test Series for UPSC Preliminary Exam 2020 ) Mark Scored : 0

3. सरकार की सौर ऊजा खरीद नीित म लै क वे फर की अपे ा लू वे फर को ाथिमकता दी गयी है ।

नीचे िदए गए कू ट का उपयोग करके सही उर चु िनए :


A. केवल 1
B. केवल 2 और 3
C. 1, 2 और 3
D. उपरोत म से कोई नहीं

Your Answer :
Correct Answer : D

Answer Justification :

एक मु ख िनणय के तहत भारत म सोलर से ल के िनमाण को बढ़ावा िमलने की उमीद है । नवीन और नवीकरणीय ऊजा
मं ालय (MNRE) ने आज नई िदली म सोलर पीवी से स के वदे शी िनमाण के सबं ध म पटीकरण िदया है ।

उले खनीय है िक MNRE के कुसु म जै से अने क मु ख कायम म वदे शी आधार पर िनिमत सोलर पीवी से ल के अिनवाय
उपयोग का ावधान िकया गया है । बहरहाल, दे खा गया है िक कई िनमाता अद्ध-सं कृत सोलर पीवी से ल (लू-वे फर) का
आयात कर रहे ह और भारत म उसका पूरा िनमाण कर रहे ह , जो मानक पर खरे नहीं उतरते ।

मं ालय ने पट िकया है िक यिद अलग-अलग टु कड़ म िसिलकॉन वे फर आयात िकए जाते ह और भारत म उस कचे माल
को सोलर पीवी से ल बनाने म इते माल िकया जाता है , तो ऐसे सोलर पीवी से ल को MNRE की योजनाओं/कायम के
तहत वदे शी िनिमत सोलर पीवी से ल नहीं माना जाएगा। केवल उहीं सोलर पीवी से ल को मायता दी जाएगी िजनका
िनमाण वदे शी आधार पर भारत म िकया जाए और िजनके िनमाण म ठोस िसिलकॉन वे फर (यानी लै क-वे फर) का उपयोग
िकया जाए।

इस िनणय से भारत म मजबूत सोलर िनमाण का आधार बनने की उमीद है ।

6 अटल योित योजना (AJAY) मु य प से संबंिधत है 

A. िदयां ग जन
B. सौर ट ीट लाइट
C. MSME म नवाचार
D. उच शै िणक सं थान की ने टविकं ग

Your Answer :
Correct Answer : B

Answer Justification :

ू रा चरण
अटल योित योजना (AJY) दस

आवे दन : सौर ऊजा चिलत ट ीट लाइट

िवीय मदद : कुल लागत का 75 ितशत मं ालय ारा और बाकी सांसद िनिध से िदया जाएगा।

लय : िनिनिलख राय/े  म कुल 3,04,500 सौर ट ीट लाइट लगाई जाएं गी :

https://telegram.me/UpscPdfDrive
https://telegram.me/UPSC_PDF
Total Marks : 200
Online Prelims Full Length Test - 4
( InsightsIAS Mock Test Series for UPSC Preliminary Exam 2020 ) Mark Scored : 0

योजना के पहले चरण के तहत उर दे श, िबहार, झारखं ड, ओिडशा और असम म काम िकया जाएगा यिक इन
राय की ओर से अितिरत मां ग है ।
पवतीय राय/ केद शािसत दे श जमू-कमीर, िहमाचल दे श और उराखं ड।
िसिकम सिहत पूवोतर के राय।
अं डमान िनकाबार ीप समूह तथा लीप। .
उपरोत पहले और तीसरे भाग म आने वाले राय को छोड़कर अय राय के 48 आकां ी िजल म ।

कायावयन ढांचा : पिरयोजना को ईईएसएल ारा कायािवत िकया जाना तािवत है । सं बंिधत सं सदीय े  के माननीय
सांसद इसके िलए तािवत थान के साथ सहमित प दान कर गे । सांसद कोष से इसके िलए रािश की मं जरू ी सं बंिधत
े  के िजलािधकारी ारा की जाएगी।

उपलिधयां : इस योजना के थम चरण के दौरान, 96 सं सदीय े  के िलए सांसद िनिध से धन के आवं टन की मं जरू ी।
वीकृत 1.45 लाख सौर ट ीट लाइट म से 1.34 लाख लगाए गए ह ।

वतमान िथित : योजना के दस ू रे चरण पर काम हो रहा है । कुल 120 सांसद की ओर सवे 131586 सौर ऊजा चिलत ट ीट
लाइन लगाए जाने की सहमित िमली है । िजसके आधार पर िजलािधकिरय की ओर से 31426 ऐसी लाइट लगाए जाने की
मं जरू ी दी गई है । अबतक ऐसे 13583 लाइट लगाई जा चु की ह ।

7 ‘ककरे खा’ िननिलिखत म से िकस सागर से होकर गुजरती है ?


1. लाल सागर
2. भूमय सागर
3. काला सागर

नीचे िदए गए कू ट का उपयोग करके सही उर चु िनए :


A. केवल 1
B. केवल 1 और 2
C. केवल 2
D. केवल 2 और 3

Your Answer :
Correct Answer : A

Answer Justification :

जै सा िक नीचे िदए गए मानिच म पट ह , कक रे खा, लाल सागर और फारस की खाड़ी से होकर गु जरती है , ले िकन काला
सागर या लाल सागर या भूमय सागर से नहीं गु जरती है ।

https://telegram.me/UpscPdfDrive
https://telegram.me/UPSC_PDF
Total Marks : 200
Online Prelims Full Length Test - 4
( InsightsIAS Mock Test Series for UPSC Preliminary Exam 2020 ) Mark Scored : 0

8 हिरत ऊजा गिलयारा (Green Energy Corridor) पिरयोजना का उे य अय ोतों से उपािदत िवु त् को िननिलिखत
म िकस के साथ संकालन (synchronize) करना है 

A. अं तराट ीय पावर िग ड


B. पारं पिरक पावर टे शन
C. ऑफ-िग ड पावर टे शन
D. (a) और (c) दोन

Your Answer :
Correct Answer : B

Answer Justification :

हिरत ऊजा गिलयारा (Green Energy Corridor) पिरयोजना का उे य िग ड म पारं पिरक िबजली टे शन के साथ, अय
ोत, जै से िक सौर एवं पवन ऊजा से उपािदत िबजली को िसं नाइज़ करना है ।

इस पिरयोजना के अं तगत अं तरायीय ट ास ं िमशन णाली आठ नवीकरणीय ऊजा समृ  राय (तिमलनाडु, राजथान,
कनाटक, आं ध दे श, महाराट ,गु जरात, िहमाचल दे श और मय दे श) ारा लागू की जा रही है ।

पिरयोजना की कुल लागत 10141 करोड़ पए है । 20 ितशत िहसे दारी राय की जबिक भारत सरकार की ओर से 40
फीसदी अनु दान िदया जा रहा है

https://mnre.gov.in/green-energy-corridor/

9 िननिलिखत म से िकन यु ों को िनणा यक माना जा सकता है िजसके पचात ईट इंिडया कं पनी भारत म एक राजनीितक शित
बन गई?

A. 1757 म लासी का यु  और 1764 म बसर की लड़ाई


B. 1762 म वांडीवाश का यु  और 1785 म पानीपत का यु 
C. 1754 म सूरत का यु  और 1772 म तूतीकोिरन की लड़ाई
D. 1772 म तूतीकोिरन की लड़ाई और 1757 म लासी की लड़ाई

Your Answer :
Correct Answer : A

Answer Justification :

बसर का यु  22 अटू बर 1764 म बसर नगर के आसपास ईट इं िडया कंपनी के है टर मु नरो और मु गल सम ाट तथा
नवाब की सं युत से नाओं के बीच लड़ा गया था।

बं गाल के नबाब मीर कािसम, अवध के नबाब शु जाउौला, तथा मु गल बादशाह शाह आलम द्िवतीय की सं युत से ना
अं गेज कंपनी से लड़ रही थी। लड़ाई म अं गेज की जीत हुई।
इस यु  के पिरणामवप 1765 म इलाहाबाद सं िध पर हतार िकए गए। मीर कािसम की हार के साथ, नवाब का
शासन समात हो गया।
पिचम बं गाल, िबहार, झारखं ड, उड़ीसा और बां लादे श का दीवानी और राजव अिधकार अं गेज कंपनी के हाथ चला

https://telegram.me/UpscPdfDrive
https://telegram.me/UPSC_PDF
Total Marks : 200
Online Prelims Full Length Test - 4
( InsightsIAS Mock Test Series for UPSC Preliminary Exam 2020 ) Mark Scored : 0

गया।

1757 ई. म हुआ लासी का यु  ऐसा यु  था िजसने भारत म अं गेज की सा की थापना कर दी। लासी का यु  बं गाल के
नवाब िसराजु ौला और ईट इं िडया कंपनी के सं घष का पिरणाम था।

लासी की लड़ाई को औपिनवे िशक शितय ारा भारतीय उपमहाीप के िनयं ण म िनणायक लड़ाई म से एक माना जाता है ।

इस यु  के अयं त महवपूण तथा थाई पिरणाम िनकले ।

यु  के बाद, अं गेज ने नवाब पर अयिधक भाव डाला और इसके पिरणामवप यापार से िपछले नु कसान और
राजव के िलए महवपूण िरयायत हािसल कीं।
अं गेज ने आगे चलकर इस राजव का उपयोग अपनी सै य शित बढ़ाने और अय यूरोपीय औपिनवे िशक शितय
जै से िक डच और ांसीसी को दिण एिशया से बाहर धकेलने के िलए िकया, इस कार िबिटश साम ाय का िवतार
हुआ।

10 जीववाद (Animism) है :

A. सभी ाकृितक शितय तथा इकाईय म आमा के अिसव का आयािमक िवचार


B. शै व तं  म मृ तक का अनु ठान
C. पिव जनजाितय का एक समु दाय
D. जं तुजगत का सामूिहक यवहार

Your Answer :
Correct Answer : A

Answer Justification :

जीववाद या सवामवाद (Animism) वह दाशिनक, धािमक या आयािमक िवचार है िक आमा न केवल मनु य म होती है
वरन् सभी जतु ओं, वनपितय, चट् टान, ाकृितक पिरघटनाओं (िबजली, वषा आिद) म भी होती है ।

इससे भी आगे जाकर कभी-कभी शद, नाम, उपमाओं, पक आिद म भी आमा के अितव की बात कही जाती है ।

सवामवाद का दशन मु यतया आिदवासी समाज म पाया जाता है परतु यह िशतो एवं िहदुओं के कुछ सदाय म भी
पाया जाता है । यह वतमान म भारत म जनजाितय समु दाय म यापक प से चिलत है तथा यह था हड़पा सयता से
जु डी हुई है ।

11 ‘राटीय ग ाम वराज अिभयान’ (RGSA) कुछ समय पहले आरभ िकया गया था। इसके बारे म िननिलिखत पर िवचार
कीिजए :
1. • इसे सतत िवकास लय (SDG) को ात करने हे तु पं चायती राज सं थान (PRI) को मजबूत करने के मु य उे य के
साथ थािपत िकया गया था।
2. • यह योजना सं िवधान के भाग- IX ारा आरित े , जहाँ पं चायती राज यवथा लागू नहीं ह , सिहत सभी राय और
कदशािसत दे श म िवतृ त है ।

नीचे िदए गए कू ट का उपयोग करके सही उर चु िनए :


A. केवल 1
B. केवल 2
C. 1 और 2 दोन

https://telegram.me/UpscPdfDrive
https://telegram.me/UPSC_PDF
Total Marks : 200
Online Prelims Full Length Test - 4
( InsightsIAS Mock Test Series for UPSC Preliminary Exam 2020 ) Mark Scored : 0

D. उपरोत म से कोई नहीं

Your Answer :
Correct Answer : C

Answer Justification :

सरकार ने 24/04/2018 को राट ीय पं चायती राज िदवस के अवसर पर राट ीय ग ाम वराज अिभयान (RGSA) की पु नगिठत
कद ायोिजत योजना (CSS) को 01.04.2018 से 31.04.2019 तक लागू करने के िलए िमशन अं योदय के साथ अिभसरण पर
मु य जोर दे ने के साथ 117 आकां ी िजल म पं चायती राज सं थाओं (PRI) को मजबूत करने पर बल दे कर सतत िवकास
लय को ात करने के िलए पं चायत को मजबूत बनाने के ाथिमक उे य के साथ 31.04.2022 को ारं भ िकया। ।

इस योजना को 7255.50 करोड़ पये के कुल बजट पिरयय के साथ अनु मोिदत िकया गया है िजसम से राय िहसे दारी
2755.50 करोड़ पये होगी और कद की िहसे दारी 4500.00 करोड़ पये होगी।यह योजना गै र-भाग IX े  सिहत सभी
राय और कदशािसत दे श तक िवतािरत है जहाँ पं चायत मौजूद नहीं ह ।

इस योजना म कदीय और राय घटक शािमल ह ।

कदीय घटक म

1. राट ीय तर की गितिविधयाँ शािमल ह जै से एनपीएमयू सिहत तकनीकी सहायता के िलए राट ीय योजना
(एनपीटीए), PRI के िलए मता िनमाण और िशण की िविभन गितिविधय के िलए एनआईआरडी और पीआर
है दराबाद के शै िणक सं थान / उकृट सं थान के साथ सहयोग
2. ई- पं चायत पर िमशन मोड ोजे ट (एमएमपी) और
3. पं चायत का ोसाहनीकरण।

राय घटक मता िनमाण और िशण के िलए राय सरकार ारा की जाने वाली गितिविधय और पं चायत की मता
बढ़ाने के िलए अय गितिविधयाँ से सं बंिधत है । राय घटक के िलए साझाकरण पै टन पूवोर और पहाड़ी राय को
छोड़कर 60:40 के अनु पात म है जहां कदीय और राय साझाकरण 90:10 के अनु पात म है । सभी सं घ राय े  के िलए,
कदीय िहसे दारी 100% है ।

इस योजना के कायम का लय चरणब सं तृित मोड के िलए पं चायत के िनवािचत ितिनिधय हे तु उनके चु नाव के छह
महीने के भीतर बु िनयादी अिभमु खी िशण और 2 साल के भीतर पु नचया िशण को सु िनिचत करना है ।

RGSA को साथक, ठोस और पिरणामोमु खी तरीके से लागू करने के िलए PRI को सम बनाने हे तु, RGSA के कायावयन के
िलए एक परे खा / ेमवक तै यार िकया गया है और राय के साथ साझा की गई है ।

https://www.panchayat.gov.in/rgsa

12 हपी का भय थापय िननिलिखत म िकस नदी बे िसन म िथत है ?

A. गोदावरी-वै गाई बे िसन


B. कावे री-पे ना बे िसन
C. कृणा-तु ं गभदा बे िसन
D. महानदी-भीमा बे िसन

Your Answer :

https://telegram.me/UpscPdfDrive
https://telegram.me/UPSC_PDF
Total Marks : 200
Online Prelims Full Length Test - 4
( InsightsIAS Mock Test Series for UPSC Preliminary Exam 2020 ) Mark Scored : 0

Correct Answer : C

Answer Justification :

हपी, कृणा-तु ं गभदा बे िसन म िथत है , इसे वष 1336 म थािपत िकया गया था तथा यह मयकालीन िहं द ू राय िवजयनगर
साम ाय की राजधानी थी।

हपी के भय खं डहर एक परकोटे -दार नगर के बारे म बताते ह । इसकी दीवार के िनमाण म िकसी सीम ट आिद का उपयोग
नहीं िकया गया था तथा इसे इं टरलॉिकंग तकनीक ारा एक साथ िनिमत िकया गया था।

1565 म िवजयनगर की हार के बाद हपी को दकनी सु तान - गोलकुंडा, बीजापु र, अहमदनगर, बरार और बीदर के शासक
ारा नट कर िदया गया।

13 पंचायत (अनुसिू चत े ों के िलए िवतार) अिधिनयम, (PESA)1996 के बारे म िननिलिखत कथनों पर िवचार कीिजए :
1. जनजातीय े  के पारं पिरक कानून, रीित-िरवाज और धािमक मायताओं के अनु सार िववाद हल सं बंिधत पं चायती राज
सं थाओं (PRI) के अिधकार े  म आते ह ।
2. PESA के अं तगत आने वाले े  म PRI के अय के सभी पद जनजातीय समु दाय के िलए आरित होते ह तथा केवल
जनजातीय समु दाय से सं बंिधत यित ही इन पद के िलए चु नाव लड़ सकते ह ।

नीचे िदए गए कू ट का उपयोग करके सही उर चु िनए :


A. केवल 1
B. केवल 2
C. 1 और 2 दोन
D. उपरोत म से कोई नहीं

Your Answer :
Correct Answer : C

Answer Justification :

पे सा या है 

यह एक कदीय कानून है , जो सं िवधान के नौव भाग म िदए गए पं चायत के ावधान को कुछ सं शोधन और छट ू के साथ
पांचवीं अनु सच
ू ी के े  के िलए िवतािरत करता है । इन े  म जनजातीय आबादी की धानता है । यह अिधिनयम
"पं चायत के ावधान का (अनु सिू चत े  के िलए िवतार) अिधिनयम, 1996 "कहा जाता है । सं ित प म इसे ''पे सा''
कहते ह ।

PESA के अं तगत कुल दस राय आते ह । ये राय ह : आं ध दे श, छीसगढ़, गु जरात, िहमाचल दे श, झारखं ड, मय
दे श, महाराट , ओिडशा, राजथान और ते लंगाना।

PESA के अं तगत आने वाले े  म PRI के अय के सभी पद जनजातीय समु दाय के िलए आरित होते ह तथा केवल
जनजातीय समु दाय से सं बंिधत यित ही इन पद के िलए चु नाव लड़ सकते ह ।

PESA के अं तगत ग ाम सभाओं को िननिलिखत तीन तरह की शितयां दान की गई ह :

1. िवकासामक : भूिम अिधग हण से पहले परामश, भूिम अलगाव को रोकना, िनषे ध लागू करने की शित, सभी
िवकासामक पिरयोजनाओं की पूव वीकृित और आिदवासी उप-योजना पर िनयं ण, िवकासामक यय के िलए

https://telegram.me/UpscPdfDrive
https://telegram.me/UPSC_PDF
Total Marks : 200
Online Prelims Full Length Test - 4
( InsightsIAS Mock Test Series for UPSC Preliminary Exam 2020 ) Mark Scored : 0

उपयोग माण प जारी करने की शित, गरीबी उमूलन के लाभािथय का चयन और अय यितगत लाभ की
योजनाएं , सामािजक े  के सं थान और अिधकािरय पर िनयं ण।
2. पारंपिरक कानूनों और रीित-िरवाजों के अनुसार िववाद का हल : आिदवासी े  के रीित-िरवाज, पारं पिरक कानून
और धािमक मायताओं के आधार पर िववाद का सामूिहक प से हल।
3. ाकृितक सं साधन का वािमव और बं धन : जल सं साधन, सामाय भूिम, लघु वनोपज, लघु खिनज आिद पर
थानीय जनजातीय समु दाय के वािमव को बनाए रखने के साथ-साथ सं बंिधत कानून के भावी कायावयन और
िनगरानी।

https://www.panchayat.gov.in/hi/pesa

14 यह भारत म सबसे बड़ा तटीय लै गन


ू है तथा भारतीय उप-महाीप म वासी पियों के िलए सबसे बड़ा शीतकालीन वास-
थल है । रामसर अिभसमय के अंतग त अंतरा टीय महव की आद भ
् िू म है । उपरोत िववरण िननिलिखत म से िकस झील का
है 

A. वे बनाड झील
B. िचका झील
C. कोले  झील
D. सु तानपु र झील

Your Answer :
Correct Answer : B

Answer Justification :

1981 म , िचका झील को रामसर अिभसमय के अं तगत अं तराट ीय महव की पहली भारतीय आदभिू म के प म नािमत
िकया गया था, इसकी समृ  जै व िविवधता के कारण तय ारा िदखाया गया है :

यहां ितवष एक लाख से अिधक वासी जलपी सिदय के मौसम आते ह । इस े  म कई दुलभ और लु ताय
जाितयां पाई जाती ह ।
एक वारनदमु खी लै गन ू के प म , यह समु दी, खारे और मीठे पानी की जाितय के अदभु त सं योजन का भरण पोषण
करता है ।
लघु शै वाल, समु दी घास, समु दी बीज, मछिलयाँ , झींगे, केकणे आिद िचका झील के खारे जल म फलते फू लते ह ।
जै व िविवधता सरण म यह झील महवपूण भूिमका िनभाती है ।

15 सुशासन सूचकांक (Good Governance Index- GGI), सभी रायों म गवनस ं की ताजा िथित का आकलन करने और
राय सरकारों एवं केद शािसत दे शों ारा उठाये गये िविभन अहम कदमों के भावों से अवगत होने का एकसमान साधन
(टूल) है । GGI 10 से टरों को यान म रखा जाताह , इसम सिमिलत है :
1. कृिष और सं ब े 
2. वािणय और उोग
3. मानव सं साधन िवकास
4. सावजिनक वाय
5. पयावरण

नीचे िदए गए कू ट का उपयोग करके सही उर चु िनए :


A. केवल 3, 4 और 5
B. केवल 1, 2 और 4
C. केवल 5

https://telegram.me/UpscPdfDrive
https://telegram.me/UPSC_PDF
Total Marks : 200
Online Prelims Full Length Test - 4
( InsightsIAS Mock Test Series for UPSC Preliminary Exam 2020 ) Mark Scored : 0

D. 1, 2, 3, 4 और 5

Your Answer :
Correct Answer : D

Answer Justification :

सभी राय एवं केद शािसत दे श म गवन स की ताजा िथित की तु लना करने के िलए मापने योय डे टा उपलध कराना,
गवन स म बे हतरी सु िनिचत करने के िलए उपयु त रणनीितय को तै यार एवं कायािवत करने म राय एवं केद शािसत
दे श को सम बनाना और पिरणाम उमु ख अवधारणाओं एवं शासन की ओर अग सर होना जीजीआई के उे य म
शािमल ह ।

‘सु शासन सूचकांक’ दरअसल सभी राय म गवन स की ताजा िथित का आकलन करने और राय सरकार एवं केद शािसत
दे श ारा उठाये गये िविभन अहम कदम के भाव से अवगत होने का एकसमान साधन (टू ल) है ।

सं केतक का चयन करते समय िविभन िसात को यान म रखा गया है । इस सं बंध म िहतधारक के साथ िविभन परामश
बै ठक आयोिजत की ग, िजनम े वार िवशे ष, मं ालय, राय एवं केद शािसत दे श के साथ सलाह-मशिवरा करना
भी शािमल ह ।

सु शासन सूचकांक म इन 10 से टर को यान म रखा जाता ह :

1. कृिष एवं सं ब े ,
2. वािणय एवं उोग,
3. मानव सं साधन िवकास,
4. सावजिनक वाय,
5. सावजिनक अवसं रचना व उपयोिगताएं ,
6. आिथक गवन स,
7. समाज कयाण एवं िवकास,
8. याियक व सावजिनक सु रा,
9. पयावरण और
10. नागिरक-केिदत गवन स।

इन सभी 10 गवन स े  या से टर को कुल िमलाकर 50 सं केतक पर मापा जाता है । सं बंिधत मूय की गणना के िलए
िकसी भी गवन स से टर के तहत िविभन सं केतक को अलग-अलग भारांक (वे टेज) िदया जाता है ।

कृिष एवं सं ब े  म िविभन वे टेज वाले 6 सं केतक ह , यथा : कृिष एवं सं ब े  की वृ दि् ध दर (0.4), खाान उपादन
की वृ दि् ध दर (0.1), बागवानी उपज की वृ दि् ध दर (0.1), दधू उपादन की वृ दि् ध दर (0.1), मांस उपादन की वृ दि् ध दर (0.1)
और फसल बीमा (0.2)।

राय और केद शािसत दे श को इन तीन समूह म िवभािजत िकया जाता है : क) बड़े राय, ख) पूवोर एवं पहाड़ी राय
और ग) केद शािसत दे श।

राय और केद शािसत दे श की र िकंग सभी सं केतक पर अलग-अलग की जाती है । इसके साथ ही इन सं केतक के
आधार पर सं बंिधत समूह के तहत इन राय एवं केद शािसत दे श के िलए सं योिजत र िकंग की भी गणना की जाती है ।

https://pib.gov.in/newsite/PrintRelease.aspx?relid=196119

16 ‘शितयों के पृ थकरण (िवतरण)’ का िसांत िननिलिखत म से िकसके ारा िदया गया था

https://telegram.me/UpscPdfDrive
https://telegram.me/UPSC_PDF
Total Marks : 200
Online Prelims Full Length Test - 4
( InsightsIAS Mock Test Series for UPSC Preliminary Exam 2020 ) Mark Scored : 0

A. जॉन रॉस (John Rawls)


B. मटे यू (Montesquieu)
C. बनाड शॉ
D. बट ड रसे ल (Bertrand Russell)

Your Answer :
Correct Answer : B

Answer Justification :

‘शितय के पृ थकरण (िवतरण)’ का िसांत, अिधकां श लोकतं  म चिलत है , तथा इसका औपिनवे िशक यु ग के दौरान भी
चलन था।

जॉज बालो (George Barlow) के सहयोग से , कॉनवॉिलस ने िबिटश भारत म शासन, याियक, पु िलस, वािणियक तथा
राजकोष के सपूण े  को कवर करते हुए एक यापक कू ट तै यार िकया।

यह सं िहता 18 वीं शतादी म पिचमी जगत म लोकिय मटे सयू के िसांत ‘सहितय का पृ थकरण’ पर आधािरत थी।

17 ‘साक सदय दे शों के मय ऊजा सहयोग (िवु त) समझौते ’ के बारे म िननिलिखत कथनों पर िवचार कीिजए :
1. इस समझौते से साक सदय राय की अिधकृत इकाइयाँ , िजनम िनजी या सावजिनक िबजली उपादक शािमल ह , े  के
भीतर िबजली खरीदने और बे चने की अनु मित दे ती ह ।
2. समझौते के अं तगत, साक सदय राट अपने -अपने दे श म े ीय सं चरण िग ड को गै र-भे दभावपूण पहुंच दान कर गे ।
3. समझौते म कहा गया है िक थाई मयथता यायालय, हे ग, सदय दे श के बीच ऊजा िविनमय और यापार से सं बंिधत
िववाद को िनपटाने का अं ितम ािधकरण होगा।

नीचे िदए गए कू ट का उपयोग करके सही उर चु िनए :


A. केवल 1
B. केवल 2 और 3
C. केवल 1 और 2
D. 1, 2 और 3

Your Answer :
Correct Answer : C

Answer Justification :

‘साक सदय दे श के मय ऊजा सहयोग (िवु त) समझौते ’ पर भारत सिहत साक के सभी सदय राय ारा हतार िकए
गए। इस समझौते से साक सदय दे श के बीच िवु त े  म और अिधक सहयोग िकया जा सकेगा।

समझौते से समूचे साक े  म िवु त आपूित म सु धार आएगा। इसके अलावा समझौते से े ीय िवु त िग ड के समे िकत
चालन म भी मदद िमले गी।

समझौते के उे य इस कार ह :

समझौता अनु मित दे ता है :

साक सदय राय की ािधकृत इकाइयाँ , िजनम िनजी या सावजिनक िबजली उपादक, िबजली उपयोिगताओं,

https://telegram.me/UpscPdfDrive
https://telegram.me/UPSC_PDF
Total Marks : 200
Online Prelims Full Length Test - 4
( InsightsIAS Mock Test Series for UPSC Preliminary Exam 2020 ) Mark Scored : 0

यापािरक कंपिनय, ट ास
ं िमशन उपयोिगताओं, िवतरण कंपिनय या कानून के तहत थािपत अय सं थान शािमल ह ,
इस े  के भीतर िबजली खरीदने और बे चने के िलए ;
सं बंिधत सदय राय के कानून ारा िविनयमन के अधीन, िनयम और शतो,ं भु गतान सु रा तं  और िबजली
यापार के कायकाल की बातचीत के िलए सं थाओं को खरीदना और बे चना ; तथा
सदय, राय को िनयात और आयात शु क म छट ू दे ने पर िवचार करते ह , और सीमा पार यापार के िलए अय
शु क और खरीदने और बे चने वाली सं थाओं के बीच िबजली का आदान-दान करते ह ।
समझौते के तहत, साक सदय राट अपने -अपने दे श म े ीय सं चरण िग ड को गै र-भे दभावपूण पहुंच दान कर गे ।
द्िवपीय - िपीय, या े ीय समझौत के मायम से - सीमा पार से िबजली यापार के िलए सीमा पार सं चरण
लाइन और सीमा पार से सं पक थािपत करने म सदय राय के बीच आयोिजत िकया जाएगा।
समझौते म एक णाली सं चालन और िनपटान तं , एक िनयामक तं  और एक िववाद िनपटान तं  बनाने की
पिरकपना की गई थी। साक मयथता पिरषद सदय राट  के बीच ऊजा िविनमय और यापार से सं बंिधत िववाद
को िनपटाने का अं ितम अिधकार है ।

https://powermin.nic.in/en/content/saarc-framework-agreement-energy-cooperation-electricity

18 तेलंगाना राय िननिलिखत म से िकन रायों के साथ सीमा बनाता है 


1. मय दे श
2. ओिडशा
3. कनाटक
4. छीसगढ़

नीचे िदए गए कू ट का उपयोग करके सही उर चु िनए :


A. केवल 3
B. केवल 3 और 4
C. 1, 2, 3 और 4
D. केवल 2 और 4

Your Answer :
Correct Answer : B

Answer Justification :

कृपया नीचे िदए गए मानिच को दे ख:

https://telegram.me/UpscPdfDrive
https://telegram.me/UPSC_PDF
Total Marks : 200
Online Prelims Full Length Test - 4
( InsightsIAS Mock Test Series for UPSC Preliminary Exam 2020 ) Mark Scored : 0

19 भारत के राटीय सु परकं यूिटं ग िमशन (NSM) के अनु योग े ों म सिमिलत ह :
1. मौसम की भिवयवाणी
2. एयरोपे स इं जीिनयिरं ग
3. िव
4. भूकंपीय िवले षण
5. आपदा बं धन

नीचे िदए गए कू ट का उपयोग करके सही उर चु िनए :


A. केवल 1, 2 और 4
B. 1, 2, 3, 4 और 5
C. केवल 3, 4 और 5
D. केवल 1, 2 और 3

Your Answer :
Correct Answer : B

Answer Justification :

राट ीय सु परकंयूिटं ग िमशन (NSM) को सात वषों की अविध म 4500 करोड़ पये की अनु मािनत लागत पर िमशन को
िवान और ौोिगकी िवभाग (DST) और इले ट ॉिनस और सूचना ौोिगकी िवभाग (DEITY) ारा सं युत प से
लागू िकया जाएगा।

उे य :

भारत को सु परकंयूिटं ग म िवव के ने तृव कताओं म से एक बनाने के िलए तथा राट ीय और वै िवक ासं िगकता की
चु नौती को हल करने म भारत की मता बढ़ाने के िलए
वै ािनक और शोधकताओं को अयाधु िनक सु परकंयूिटं ग सु िवधाओं दान करना तथा उह अपने सं बंिधत डोमे न म
अयाधु िनक अनु संधान करने म सम बनाना।

https://telegram.me/UpscPdfDrive
https://telegram.me/UPSC_PDF
Total Marks : 200
Online Prelims Full Length Test - 4
( InsightsIAS Mock Test Series for UPSC Preliminary Exam 2020 ) Mark Scored : 0

अितरे क और यास के दोहराव को कम करने और सु परकंयूिटं ग म िनवे श का अनु कूलन करने के िलए
वै िवक ितपधा हािसल करना और सु परकंयूिटं ग तकनीक के रणनीितक े  म आमिनभरता सु िनिचत करना

उपये ाग े :

जलवायु पिरवतन ; मौसम की भिवयवाणी ; एयरोपे स इं जीिनयिरं ग ; कयूटेशनल बायोलॉजी ; आणिवक


गितशीलता ; परमाणु ऊजा िसमु लेशन ; राट ीय सु रा / रा अनु योग ; भूकंपीय िवले षण ; आपदा िसमु लेशन और
बं धन
कयूटेशनल रसायन िवान ; कयूटेशनल सामग ी िवान और नै नोमटे िरयस ; पृ वी (खगोल भौितकी) से परे की
खोज ; बड़े जिटल िसटम िसमु लेशन और साइबर िफिजकल िसटम ; िबग डे टा एनािलिटस ; िव; सूचना
िरपोिजटरी / सरकारी सूचना णाली

https://nsmindia.in/about

20 िननिलिखत कथनों पर िवचार कीिजए :


1. आिथक वतं ता सूचकांक का काशन ‘UNESCAP’ ारा िकया जाता है ।
2. ‘लोबल इकोनॉिमक ीडम इं डेस’ मै स ल क इं टीटूट ारा िनिमत िकया गया है ।
3. आिथक वतं ता के इन दोन वै िवक सूचकांक म भारत का थान िनचले म म है ।

नीचे िदए गए कू ट का उपयोग करके सही उर चु िनए :


A. केवल 2
B. केवल 1 और 3
C. केवल 3
D. 1, 2 और 3

Your Answer :
Correct Answer : C

Answer Justification :

‘आिथक वतं ता सूचकांक’ िवव के िविभन दे श म दी गई आिथक वतं ताओं का तु लनामक अययन करने का
बहुआयामी सूचकांक है । आिथक वतं ता के वै िवक सूचकांक म भारत का थान िनचले पायदान पर है ।

आिथक वतं ता सूचकांक, हे िरटे ज फाउं डेशन ारा जारी िकया जाता है तथा लोबल इकोनॉिमक ीडम इं डेस को ‘ेजर
इं टीटूट’ ारा कािशत िकया जाता है ।

आिथक वतं ता सूचकांक म भारत की रै िकंग उसके पड़ोसी मु क पािकतान समे त कई दिण एिशयाई दे श से भी पीछे है ।
इस सूचकांक म भारत ने कुल 52.6 अं क हािसल िकये जो िपछले साल के मु काबले 3.6 अं क कम है । िपछले साल इस सूचकांक
म भारत की र िकंग 123 थी। इस सूचकांक म हॉगकॉग, िसं गापु र और यूजील ड शीष पर रहे ह ।

21 आवयक वतु ओं या उपादों के उपादन, आपूित व िवतरण तथा यापार को िनयंित करने के िलये आवयक वतु
अिधिनयम, 1955 बनाया गया था। इसके बारे म िननिलिखत कथनों पर िवचार कीिजए :
1. वतु ओं की भं डार सीमा को िविनयिमत करने तथा माल की आवाजाही को ितबं िधत करने की शितयां केवल कद सरकार के
पास ह ।
2. बीज और उवरक ECA के दायरे से बाहर ह ।
3. खा एवं नागिरक आपूित ािधकरण राय म अिधिनयम के ावधान को िनपािदत कर सकते ह ।

https://telegram.me/UpscPdfDrive
https://telegram.me/UPSC_PDF
Total Marks : 200
Online Prelims Full Length Test - 4
( InsightsIAS Mock Test Series for UPSC Preliminary Exam 2020 ) Mark Scored : 0

नीचे िदए गए कू ट का उपयोग करके सही उर चु िनए :


A. केवल 1
B. केवल 2 और 3
C. केवल 3
D. 1, 2 और 3

Your Answer :
Correct Answer : C

Answer Justification :

आवयक वतु अिधिनयम, 1955 िनयम और िविनयम का िनधारण नहीं करता है , बिक राय को डीलर लाइस िसं ग से
सं बंिधत िनयं ण आदे श जारी करने , टॉक सीमा को िविनयिमत करने , माल की आवाजाही को ितबं िधत करने और ले वी की
णाली के तहत अिनवाय खरीद की आवयकताओं को ितबं िधत करने की अनु मित दे ता है ।

अिधिनयम. जत िकए गए टॉक को जत करने की कारवाई के िलए भी ावधान करता है । अिधिनयम के अं तगत कद
सरकार ारा अिधकां श शितयां राय सरकार को इस िदशा म सप दी गई ह िक वे इन शितय का योग कर गे । राय म
खा और नागिरक आपूित ािधकरण अिधिनयम के ावधान को िनपािदत करते ह ।

अिधिनयम म मु ख कमोिडटी समूह सिमिलत ह :

1. पे टोिलयम और उसके उपाद, िजनम पे टोल, डीजल, केरोिसन, ने था, सॉव ट् स आिद ;
2. खा ते ल, बीज, वनपित ते ल, दाल , गना और इसके उपाद जै से, खांडसारी और चीनी, चावल धान ;
3. कचा जूट और जूट व;
4. डस- आवयक दवाओं की कीमत अभी भी डीपीसीओ ारा िनयं ित ह ;
5. उवरक- उवरक िनयं ण आदे श कीमत के अलावा उवरक के हतांतरण और टॉक पर ितबं ध लगाता है ;
6. याज और आलू ;
7. खा फसल, फल और सिजय, मवे िशय के चारे , जूट के बीज और कपास के बीज।

22 संिवधान म रायों और कद शािसत दे शों की सूची िननिलिखत म िकस अनुसच
ू ी के अंतग त दी गयी है 

A. पहली अनु सच ू ी
B. ू
दसरी अनु सचू ी
C. तीसरी अनु सच ू ी
D. दसवीं अनु सचू ी

Your Answer :
Correct Answer : A

Answer Justification :

ये क अनु सच
ू ी म शािमल िवषय ह :

पहली अनु सच ू ी : राय और कद शािसत दे श की सूची


दसू री अनु सच
ू ी : राट पित, रायपाल, मु य यायाधीश, उच यायालय और उचतम यायालय के यायाधीश,
िनयं क और महाले खा परीक का वे तन
तीसरी अनु सच ू ी : शपथ एवं ितान के ितप
चौथी अनु सच ू ी : राय सभा म भारत के ये क राय के िलए सीट आवं टन

https://telegram.me/UpscPdfDrive
https://telegram.me/UPSC_PDF
Total Marks : 200
Online Prelims Full Length Test - 4
( InsightsIAS Mock Test Series for UPSC Preliminary Exam 2020 ) Mark Scored : 0

पांचवीं अनु सचू ी : अनु सिू चत े  और जनजाितय का शासन और िनयं ण


छठी अनु सच ू ी : असम, मे घालय, िपु रा, िमजोरम और अणाचल दे श म जनजातीय े  के शासन के िलए
ावधान
सातवीं अनु सच ू ी : सं घ और राय के बीच शितय और कायों का आवं टन
आठवीं अनु सच ू ी : सं िवधान ारा मायता ात भारत की 22 भाषाओं की सूची

अय 4 अनु सिू चयां , दलबदल िवरोधी, भूिम अिधग हण कानून और थानीय वशासन से सं बंिधत ह ।

23 9वीं शतादी म उर भारत पर िनयंण के िलए "िपीय संघष " के िननिलिखत म िकनके मय हुआ था

A. मौय, गु त और हषवधन
B. ितहार साम ाय, पाल साम ाय और राट कू ट साम ाय
C. सातवाहन, चालु य और पाल साम ाय
D. नं द, शु ं ग और कव

Your Answer :
Correct Answer : B

Answer Justification :

उरी भारत पर िनयं ण के िलए िपीय सं घष नौवीं शतादी म हुआ था। यह सं घष ितहार साम ाय, पाल साम ाय और
राट कू ट साम ाय के मय हुआ था।

पाल का भारत के पूवी भाग पर शासन था जबिक ितहार के िनयं ण म पिचमी भारत (अवं ती-जालौर े ) था। राट कू ट
ने भारत के दकन े  पर शासन िकया था। इन तीन राजवं श के बीच उरी भारत (कनौज) पर िनयं ण के िलए हुए सं घष
को भारतीय इितहास म िपीय सं घष के प म जाना जाता है ।

उरी भारत (कनौज) पर अिधपय को ले कर लगभग 200 वषों तक इन तीन महाशितय के बीच सं घष जारी रहा। इस
सं घष म अितम सफलता गु जर-ितहार को िमली।

24 भारत का पहला यापार समझौता िननिलिखत म से िकस समूह के साथ िकया गया था?

A. मकोसु र
B. आिसयान
C. िबसटे क
D. SAFTA

Your Answer :
Correct Answer : C

Answer Justification :

https://telegram.me/UpscPdfDrive
https://telegram.me/UPSC_PDF
Total Marks : 200
Online Prelims Full Length Test - 4
( InsightsIAS Mock Test Series for UPSC Preliminary Exam 2020 ) Mark Scored : 0

Ch 5: Economic Survey 2019-20

25 लोकसभा के िया तथा काय संचालन के िनयम 377 के संबंध म िननिलिखत कथनों पर िवचार कीिजए :
1. सदय को, ऐसे मामले जो यवथा का न नहीं ह अथवा िजन मामल को िकसी अय िनयम के अं तगत नहीं उठाया जा
सकता, िनयम 377 के अधीन उठाए जाने की अनु मित ह ।
2. िनयम 377 के तहत मामला उठाने की सूचना सं बंिधत सांसद ारा िलिखत म दी जानी आवयक है ।
3. 377 के तहत कोई भी मामला सदन के अय की अनु मित के पचात ही उठाया जा सकता है ।

नीचे िदए गए कू ट का उपयोग करके सही उर चु िनए :


A. केवल 1
B. केवल 2 और 3
C. केवल 1 और 3
D. 1, 2 और 3

Your Answer :
Correct Answer : D

Answer Justification :

सदय को ऐसे मामले जो यवथा का न नहीं ह अथवा िजन मामल को लघु सूचना न, कॉिलं ग अट शन मोशन आिद

https://telegram.me/UpscPdfDrive
https://telegram.me/UPSC_PDF
Total Marks : 200
Online Prelims Full Length Test - 4
( InsightsIAS Mock Test Series for UPSC Preliminary Exam 2020 ) Mark Scored : 0

से सं बंिधत िनयम के अं तगत नहीं उठाया जा सकता, िनयम 377 के अधीन िवशे ष उले ख ारा उठाए जाने की अनु मित होती
ह ।

िनयम 377 के तहत मामला उठाने के िलए नोिटस को िलिखत प म महासिचव को सं बोिधत िकया जाना चािहए।
नोिटस दे ने के िलए मानक मु िदत प सं सदीय सूचना कायालय म उपलध ह ।
बयान के ताव का ले ख आमतौर पर 150 शद से अिधक नहीं होना चािहए।
िनयम 377 के तहत कोई भी मामला सदन के अय की अनु मित के पचात ही उठाया जा सकता है ।

http://loksabhaph.nic.in/writereaddata/Abstract/matters_under_rule_377.pdf

26 S&P लोबल माकट इंटेिलज स िरपोट 2019 के बारे म िननिलिखत कथनों पर िवचार कीिजए :
1. वै िवक शीष 100 ब क म भारत का केवल एक ब क सिमिलत है ।
2. वै िवक शीष 100 ब क म सवािधक ब क USA की है ।

नीचे िदए गए कू ट का उपयोग करके सही उर चु िनए :।


A. केवल 1
B. केवल 2
C. 1 और 2 दोन
D. उपरोत म से कोई नहीं

Your Answer :
Correct Answer : A

Answer Justification :

िच 1 (सव ण से नीचे ) से पता चलता है िक भारत की अथयवथा के आकार की दृिट से इसके ब क अनु पाितक प से
काफी छोटे ह ।

2019 म , जबिक भारतीय अथयवथा िवव म पांचवीं सबसे बड़ी है , तो हमारा सवोच े णीात ब क - टे ट ब क ऑफ
इं िडया- िवव के 55 व थान पर जो िक काफी नीचे है और लोबल टॉप 100 म थान पाने वाला एकमा ब क है ।

वै िवक शीष 100 ब क म भारत का केवल एक ब क है और इस िवशे षता पर उन दे श के साथ समूहब है जो इसके आकार का
एक यून अं श ह : िफनल ड (लगभग 1/11 वां ), डे नमाक (1/8 वां ), नॉव (1/7 वां ), ऑिट या ( लगभग 1/7 वां ), और बे िजयम
(लगभग 1/6 वां )।

https://telegram.me/UpscPdfDrive
https://telegram.me/UPSC_PDF
Total Marks : 200
Online Prelims Full Length Test - 4
( InsightsIAS Mock Test Series for UPSC Preliminary Exam 2020 ) Mark Scored : 0

Ch 7: Economic Survey 2019-20

27 िब िटश कालीन भारत म ‘दतक’ िननिलिखत म िकससे संबंिधत था

A. आं तिरक यापार के िलए मु त पास णाली


B. ांतीय िवषय का एक आदे श
C. थानीय यापािरय का समु दाय
D. डाक िटकट

Your Answer :
Correct Answer : A

Answer Justification :

बं गाल म ईट इं िडया कपनी दतक पारप जारी करती थी, उनम कपनी के अिभकताओं को अिधकार िदया जाता था िक वे
ात के अं दर चु ं गी अदा िकये िबना यापार कर सक।

1717 ई. म शाह फ़ ख़िसयर ारा कपनी को िदये गये फ़रमान के अं तगत ढाई ितशत चु ं गी अदा न करने की छट ू दी गई
थी। क़ानूनी तौर पर यह छटू के वल कपनी को ही ात हो सकती थी। ले िकन इस छ ट
ू का बे जा फ़ायदा दो कार से उठाया
जाता था। पहले तो कपनी के कमचारी दतक ात करके वयं िबना चु ं गी िदये िनजी यापार करते थे । िफर कपनी इस
कार के दतक भारतीय यापािरय को भी बे च िदया करती थी, िजनके ारा वे लोग भी िबना चु ं गी िदए यापार करते थे ।

वारे न हे िटं स ने दातक की णाली को समात कर िदया। इन मु त पास को हटाते हुए, उहने आं तिरक यापार को
िविनयिमत करने का इरादा िकया। उसने चु ं गी घर की सं या कम कर दी और भारतीय और गै र-भारतीय सामान के िलए एक
समान टै िरफ लागू िकया।

28 िननिलिखत म से कौन सी संथाय भारत म शै डो ब िकं ग णाली का िहसा हो सकती है ?


1. गै र-ब िकंग हाउिसं ग फाइन स कंपिनयां (HFC)
2. खु दरा गै र-ब िकंग िवीय कंपिनयां (िरटे ल-NBFC)
3. तरल ऋण यु चु अल फंड (LDMF)

नीचे िदए गए कू ट का उपयोग करके सही उर चु िनए :


A. केवल 2
B. केवल 3
C. केवल 1 और 2
D. 1, 2 और 3

Your Answer :
Correct Answer : D

Answer Justification :

शै डो ब िकंग णाली (Shadow Banking System) िवीय मयथ का एक समूह है जो वै िवक िवीय णाली म साख
(ेिडट) िनमाण का काय करते ह , ले िकन इसके सदय िनयामक िनगरानी के अधीन नहीं होते ।

घोष व अय, 2012 के अनु सार ’शै डो ब िकंग म वे गितिविधयाँ , बाजार, सं िवदाएं और ऐसी सं थाएं शािमल होती ह जो

https://telegram.me/UpscPdfDrive
https://telegram.me/UPSC_PDF
Total Marks : 200
Online Prelims Full Length Test - 4
( InsightsIAS Mock Test Series for UPSC Preliminary Exam 2020 ) Mark Scored : 0

आं िशक या पूण प से पारं पिरक वािणियक ब िकंग े  के बाहर से चालन करती है या हलके प से िविनयिमत होती है या
िबलकुल नहीं होती।

शै डो ब िकंग णाली म कोई ऐसी एक इकाई हो सकती है जो िनिधय के अं ितम आपूितकताओं व अं ितम उपयोगकताओं म
मय मयथता करती है या इसम बहुत सी ं खला बढ इकाईयां शािमल हो सकती ह । शै डो ब क के पास कदीय ब क तरलता
तक य पहुंच नहीं होती है । शै डो ब िकंग णालीपर जोिखम भरी और अं तराल पिरसं िपिय का भारी दबाब है जबिक
इसकी दे यताएं ‘ब क रन’ की िथित पै दा कर सकती है ।

आिथक सव ण भारत म शै डो ब िकंग णाली के तीन महवपूण खं ड पर यान किदत करता है , अथात,् गै र-ब िकंग हाउिसं ग
फाइन स कंपनी (HFC), िरटे ल गै र-ब िकंग िवीय कंपिनयां (िरटे ल-NBFC) और िलिवड डे ट यूचुअल फंड (LDMF)।

NBFC े  को सावजिनक और िनजी े  के ब क और अय िवीय सं थान से यु त पारं पिरक ब िकंग णाली की तु लना म
हका िविनयिमत िकया जाता है । हालाँ िक, NBFC े  म िनयमन समय के साथ िवकिसत हुआ है , िजसके तहत िववे कपूण
मानदं ड ारा NBFC ारा जमा ले ने को हतोसािहत िकया गया (भारतीय िरजव ब क (आरबीआई), 1998) और जमा न ले ने
वाले NBFC (आरबीआई, 2006) को ोसािहत िकया गया है ।

इन दोन भाव के सं योजन NBFC के िवपोषण ोत म जमाओं की िहसे दारी घटी है और थोक िवपोषण म वृ दि् ध हुई
है । NBFC के थोक फंिडं ग ोत म मु य प से ब क शािमल ह (ाथिमक प से साविध ऋण के मायम से और गै र-
पिरवतनीय िडब चर और वािणियक पे पर के मायम से ) तथा ऋण यूचुअल फंड (गै र-पिरवतनीय िडब चर और वािणियक
पे पर के मायम से ) शािमल होते ह ।

29 उिरमे र िशलालेख, दसवीं शतादी ईवी का एक िस तिमल िशलालेख है । यह संबंिधत है :

A. ग ाम सभाओं की िनवाचन पित का अिभले िखन


B. उर म िकए जाने वाले बिल समारोह
C. मिहलाओं के अिधकार एवं िवरासत
D. िविध-िनमाणक िनकाय और कायकारी िनकाय का पृ थकीकरण

Your Answer :
Correct Answer : A

Answer Justification :

उिरमे र (Uthiramerur) दिण भारत म तिमलनाडु राय के कांचीपु रम िज़ले का एक पं चायती ग ाम है । चोल राय के
अं तगत बाण (अग हार) के एक बड़े ग ाम म दसवीं शतादी के पचात् अने क िशलाले ख थानीय राजनीित पर काश पर
काश डालते ह , जो इस कार ह -

ग ाम 30 भाग म िवभािजत था, िजनम से ये क भाग का ितिनिध लाटरी ारा चु नी गई वािषक पिरषद म उपिथत
होता था।
पिरषद पाँच उपसिमितय म िवभािजत थी िजनम से तीन मश : उान तथा वािटकाओं, तालाब तथा िसं चाई और
झगड़ के िनबटारे के िलए उरदायी थीं, जबिक अं ितम दो के काय अिनिचत ह ।
सदय अवै तिनक होते थे तथा दुयवहार के कारण पदयु त िकये जा सकते थे ।
पिरषद म सिमिलत होने का आिधकार सपि योयता ारा सीिमत था, िजसम मकान तथा छोटा भू-भाग
सिमिलत था।
सदयता 35 तथा 70 वष की आयु के मय वाले यितय के िलए सीिमत थी जो एक वष तक कायभार सँ भाल ले ते
थे , वे तीन वष के िलए पु न : िनयु ित हे तु अयोय हो जाते थे ।
उिरमे र िवधान की दो अं ितम िवशे षताएँ अय ग ाम के िवधान म भी पायी जाती ह , िजनकी ले खा आज भी ात

https://telegram.me/UpscPdfDrive
https://telegram.me/UPSC_PDF
Total Marks : 200
Online Prelims Full Length Test - 4
( InsightsIAS Mock Test Series for UPSC Preliminary Exam 2020 ) Mark Scored : 0

ह ।
दिण की ये पिरषद न केवल झगड़ का िनबटारा करती थीं तथा सरकार की सीमा के बाहर सामािजक कायों का बध
करती थीं, अिपतु मालगु जारी एक करने , यितगत सहयोग का मूय िनधािरत करने की बातचीत के िलए
उरदायी थीं।
गाँ व की बं जर भूिम का वािमव िवय अिधकार सिहत िनिचत प से उनका था तथा वे िसं चाई, माग-िनमाण तथा
अय जनकायों म िवशे ष िच रखते थे ।
उनके आदान दान का िववरण ग ाम के मिदर की दीवार पर अं िकत रहता था िजससे एक शितशाली सामु दाियक
जीवन का आभास िमलता है और जो ारिभक भारतीय राजनीित के सवेठ अं श के थायी मारक ह ।

30 िननिलिखत म से िकन अिधिनयमों का यावसाियक सुरा, वाय और काय िथितयाँ संिहता, 2019 म समावेशन िकया
जाना तािवत है ?
1. कारखाना अिधिनयम, 1948
2. भारतीय यापार सं घ अिधिनयम, 1926
3. कमचारी भिवय िनिध और िविवध ावधान अिधिनयम, 1952

नीचे िदए गए कू ट का उपयोग करके सही उर चु िनए :


A. केवल 1 और 3
B. केवल 2
C. 1, 2 और 3
D. केवल 1

Your Answer :
Correct Answer : D

Answer Justification :

सु रा, वाय, कयाण और बे हतर कामकाजी पिरिथितयां कामगार की भलाई के िलए और दे श के आिथक िवकास के िलए
भी आवयक ह यिक दे श का वथ कायबल अिधक उपादक होगा और दुघट  नाओं और अयािशत घटनाओं म कमी से
िनयोताओं को आिथक प से लाभ होगा।

यावसाियक सु रा, वाय और काय िथितयाँ सं िहता, 2019 दे श के सभी कायबल के िलए सु रा और वथ काय
िथितय का िवतार करने के अं ितम उे य के साथ सु रा, वाय, कयाण और काय िथितय के ावधान के
दायरे को मौजूदा 9 मु ख े  से ले कर 10 या अिधक कमचािरय वाले सभी ितठान तक बढ़ाती है ।
तािवत सं िहता िमक के कवरे ज को कई गु ना बढ़ाती है यिक यह 10 या अिधक िमक को िनयु त करने वाले
सभी उोग, यापार, यवसाय और िविनमाण ितठान पर लागू होगा। इसम आईटी े  या से वा े  के ितठान
शािमल ह ।
इसके अलावा सं िहता म खदान और बं दरगाह को छोड़कर 10 कमचािरय वाले सभी ितठान के िलए िविभन
उपयु तताओं को एक साथ िदया गया है ।
एक मा कमी वाले खदान और गोदी म यह सं िहता लागू होगी। सं िहता म यापक कवरे ज सु िनिचत करने के िलए
मजीवी पकार और िसने मा किमय की पिरभाषाओं के अं दर इले ट ॉिनक मीिडया म कायरत िमक और ऑिडयो
िवजु अल उपादन के सभी प को शािमल करने के िलए भी सं शोधन िकया गया है ।
इसी तरह, अं तर-राय वासी िमक की पिरभाषा म उन वासी िमक को शािमल करने के िलए सं शोधन का
ू रे राय के िनयोता ारा सीधे िनयोिजत िकया जा रहा है । यह
ताव है , िजह ठे केदार या एज ट के िबना दस
ताव सु रा, वाय और काय िथितय के ावधान के कवरे ज को कई गु ना बढ़ा दे गा।

https://telegram.me/UpscPdfDrive
https://telegram.me/UPSC_PDF
Total Marks : 200
Online Prelims Full Length Test - 4
( InsightsIAS Mock Test Series for UPSC Preliminary Exam 2020 ) Mark Scored : 0

13 कदीय म अिधिनयम के ासं िगक ावधान को िमलाकर, उह सरल और यु ितसं गत बनाने के बाद सं िहता का
मसौदा तै यार िकया गया है ।

इनम फैट ी कानून, 1948; खदान कानून, 1952; डॉक वक स (सु रा, वाय और कयाण अिधिनयम, 1986), भवन और
अय िनमाण िमक (रोजगार और से वा की शतों का िविनयमन) अिधिनयम, 1996, लांट िमक कानून, 1951; अनु बंध
म (िविनयमन और उमूलन) अिधिनयम, 1970; अं तर-राय वासी कामगार (रोजगार और से वा की शतों का
िविनयमन) अिधिनयम, 1979; मजीवी पकार और अय यूज़ पे पर एलाइज (से वा की शत और ावधान)
अिधिनयम, 1955; मजीवी पकार (मजदरू ी की दर का िनधारण) अिधिनयम, 1958; मोटर ट ास ं पोट वक स एट, 1961;
िबी सं वधन कमचारी (से वा की िथित) अिधिनयम, 1976; बीड़ी और िसगार िमक (रोजगार की िथितयां )
अिधिनयम, 1966 और द िसने वक स और िसने मा िथये टर वक स एट, 1981 शािमल ह ।

सं िहता लागू होने के बाद, सं िहता म शािमल िकए जा रहे सभी अिधिनयम को िनरत कर िदया जाएगा।

https://www.prsindia.org/billtrack/occupational-safety-health-and-working-conditions-code-2019

31 ािधकृत आिथ क संचालक (AEO) िननिलिखत म से िकसके तवावधान म संचािलत काय म है 

A. िवव यापार सं गठन (डयूटीओ)


B. िवव सीमा शु क सं गठन (WCO)
C. अं तराट ीय िमक सं घ (ILU)
D. सं युत राट आिथक और सामािजक पिरषद (UNECOSOC)

Your Answer :
Correct Answer : B

Answer Justification :

ािधकृत आिथक सं चालक (AEO) वै िवक यापार को सु रित और सु िवधाजनक बनाने के िलए िवव सीमा शु क सं गठन
(WCO), ेमवक ऑफ़ ट डड्स  (SAFE) के तवावधान म सं चािलत एक कायम है । इस कायम का उे य अं तराट ीय
आपूित ृं खला सु रा म वृ दि् ध करना और माल के आवागमन को सु गम बनाना है ।

AEO अं तराट ीय आपूित ृं खला म अने क कारोबारी शािमल है । इस कायम के तहत, अं तराट ीय यापार म लगे ितठान
को सीमा शु क ारा आपूित ृं खला सु रा मानक के अनु पालनकता के प म अनु मोिदत िकया जाता है तथा AEO का दजा
िदया जाता है ।

AEO दजा ात इकाई को 'सु रित' यापारी और िववसनीय यापािरक भागीदार माना जाता है । AEO एक वै िछक
कायम है ।

32 िब िटश भारत म , सवोच यायालय की थापना िननिलिखत िकस िबधान के ारा की गयी थी

A. रे यु ले िटं ग एट, 1773


B. रॉयल चाट र, 1825
C. िपट् स इं िडया एट, 1784
D. काउं िसल एट, 1813

Your Answer :

https://telegram.me/UpscPdfDrive
https://telegram.me/UPSC_PDF
Total Marks : 200
Online Prelims Full Length Test - 4
( InsightsIAS Mock Test Series for UPSC Preliminary Exam 2020 ) Mark Scored : 0

Correct Answer : A

Answer Justification :

फोट िविलयम, कलका (कोलकाता) म सवोच यायालय की थापना वष 1774 म रे गु ले िटं ग एट, 1773 के ारा की गई
थी। यह 1774 से िबिटश भारत का सवोच यायालय था िजसे बाद म भारतीय उच यायालय अिधिनयम 1861 ारा
ितथािपत िकया गया था।

रे यु ले िटं ग एट के महवपूण ावधान थे :

कोट आफ़ डायरे टर का कायकाल 1 वष के थान पर 4 वष का हो गया तथा डायरे टर की सं या 24 िनधािरत की
गयी।
कोट आफ़ े सीड सी (बं गाल) के शासक को अब अं गेज़ी े  का गवनर जनरल कहा जाने लगा तथा उसको सलाह
दे ने हे तु 4 सदय की एक कायकािरणी बनाई गयी, िजसका कायकाल 5 वष का होता था।
मदास तथा बबई के गवनर उसके अधीन हो गये । ये केवल कोट आफ़ डाये रेटस की िसफ़ािरश पर िबिटश सम ाट ारा
ही 5 वष के पूव हटाये जा सकते थे ।
कलका म एक सु ीम कोट की थापना की गयी, िजसम एक मु य यायधीश और तीन अय यायाधीश िनयु त
िकये गये , जो अं गेज़ी क़ानून के अनु सार जा के मु क़दम का िनणय करते थे ।
इसका काय े  बं गाल, िबहार, उड़ीसा तक था। इस सवोच यायालय को साय याय तथा सामाय िविध के
यायालय, नौसे ना िविध के यायालय तथा धािमक यायालय के प म काम करना था।
उचतम यायालय 1774 ई. म गिठत िकया गया और सर एलीजा इपी मु य यायाधीश तथा चे बज, िलमै टर और
हाइड अय यायाधीश िनयु त हुए।
िबना लाइस स ात िकए कपनी के कमचारी को िनजी यापार करने से ितबं िधत कर िदया गया।
गवनर जनरल व उसकी किसल को िनयम बनाने तथा अयादे श पािरत करने का अिधकार िदया गया, पर यह सु ीम
कोट ारा पं जीकृत होना ज़री था।
कपनी के ये क सै िनक अथवा असै िनक पदािधकारी को िकसी भी यित के उपहार, दान या पािरतोिषक ले ने से
ितबं िधत कर िदया गया।
कपनी के अिधकािरय व कमचािरय के वे तन को बढ़ा िदया गया।

33 िननिलिखत कथनों पर िवचार कीिजए :


1. पं चायती राज मं ालय ने सं बंिधत राय ारा कायावयन के िलए ग ाम पं चायत िवकास योजना (GPDP) के िलए मॉडल
िदशािनद श तै यार िकए ह ।
2. ग ाम पं चायत के िलए सं वैधािनक प से यह अिनवायता की गई है िक आिथक िवकास और सामािजक याय के िलए ग ाम
पं चायत िवकास योजनाएं (GPDP) तै यार कर ।

नीचे िदए गए कू ट का उपयोग करके सही उर चु िनए :


A. केवल 1
B. केवल 2
C. 1 और 2 दोन
D. कोई नहीं

Your Answer :
Correct Answer : C

Answer Justification :

14 व िव आयोग अवाड ने ग ाम पं चायत के अं ितम छोर तक सं थागत तर पर उरदायी थानीय शासन के िलए एक बड़ा

https://telegram.me/UpscPdfDrive
https://telegram.me/UPSC_PDF
Total Marks : 200
Online Prelims Full Length Test - 4
( InsightsIAS Mock Test Series for UPSC Preliminary Exam 2020 ) Mark Scored : 0

अवसर पै दा िकया है । थानीय िनकाय के िलए अनु दान की िनमु ि त और उपयोग के िलए िव मं ालय ारा जारी
िदशािनद श यह िनधािरत करते ह िक FFC अवाड के तहत यय करने से पहले राय कानून के अनु सार उह सपे गए कायों
के िलए ग ाम पं चायत ारा बु िनयादी से वाओं के िलए उिचत योजनाएं तै यार की जानी चािहए।

ग ाम पं चायत के िलए सं वैधािनक प से यह अिनवायता की गई है िक उनके पास उपलध सं साधन का उपयोग करते हुए
आिथक िवकास और सामािजक याय के िलए ग ाम पं चायत िवकास योजनाएं (GPDP) तै यार की जाएँ ।

ग ाम पं चायत िवकास योजना (GPDP) यापक होनी चािहए और समु दाय िवशे षकर ग ाम सभा को शािमल करने वाली
भागीदारी िया पर आधािरत होनी चािहए, और सं िवधान की यारहवीं अनु सचू ी म सूचीब 29 िवषय से सं बंिधत सभी
कदीय मं ालय / सं बंिधत िवभाग की योजनाओं के साथ अिभसिरत होगी।

अिभसरण इस तय को दे खते हुए अिधक महव रखता है िक ग ामीण भारत के पिरवतन के िलए राट ीय महव के िवषय से
सं बंिधत महवपूण योजनाओं के भावी कायावयन के िलए पं चायत महवपूण भूिमका िनभा सकती ह ।

इस मं ालय ने GPDP के िलए मॉडल िदशािनद श तै यार करने की पहल की है और इसे उन सभी राय म पिरचािलत िकया
जहां सं िवधान का भाग IX लागू है ।

इन मॉडल िदशािनद श के आधार पर, सभी राय ने GPDP के िलए अपने राय के िविशट िदशािनद श को अिधसूिचत
िकया है । सं बंिधत राय िदशािनद श के अनु सार, तब से GPDP को राय ारा तै यार और कायािवत िकया जा रहा था।

इसके अलावा, गां व के समग िवकास की आवयक जरत के अनु प, मं ालय ने 2015 के दौरान जारी मॉडल GPDP
िदशािनद श को सं शोिधत िकया है । सं शोिधत िदशािनद श, 2018 को मं ालय की वे बसाइट पर डाल िदया गया है और राय
के साथ उनके सं बंिधत िदशािनद श को आगे बढ़ाने के िलए साझा िकया गया है ।

https://www.panchayat.gov.in/gpdp

34 रामानुज ारा ितपािदत िविशटाै त म िननिलिखत म से िकस पर बल िदया गया है 


1. आम तव तथा कृित तव ब के ही दो वप ह ।
2. तक की अपे ा भित से मो ात होता है ।

उपरोत म से कौन सा सही है / ह 


A. केवल 1
B. केवल 2
C. 1 और 2 दोन
D. कोई नहीं

Your Answer :
Correct Answer : A

Answer Justification :

िविशटाै त, आचाय रामानु ज (सन् 1037-1137 .) का ितपािदत िकया हुआ यह दाशिनक मत है । इसके अनु सार यिप
जगत् और जीवामा दोन कायतः ब से िभन ह िफर भी वे ब से ही उदभूत ह और ब से उसका उसी कार का सं बंध है
जै सा िक िकरण का सूय से है , अतः ब एक होने पर भी अने क ह ।

इस िसांत म आिद शं कराचाय के मायावाद का खं डन है । शं कराचाय ने जगत को माया करार दे ते हुए इसे िमया बताया है ।
ले िकन रामानु ज ने अपने िसांत म यह थािपत िकया है िक जगत भी ब ने ही बनाया है । पिरणामवप यह िमया नहीं

https://telegram.me/UpscPdfDrive
https://telegram.me/UPSC_PDF
Total Marks : 200
Online Prelims Full Length Test - 4
( InsightsIAS Mock Test Series for UPSC Preliminary Exam 2020 ) Mark Scored : 0

हो सकता।

रामानु ज अपने दाशिनक दृिटकोण म अलवार सं तो से काफी भािवत थे ।


उनके अनु सार मो ात करने का सबसे अछा साधन िवणु के ित गहन समपण था।
िवणु अपनी कृपा से भत को उनके साथ िमलन का आनं द ात करने म मदद करते ह ।
िविशटाै त दशन म सा या परमसत् के सबध म तीन तर माने गये ह - ब अथात् ईवर, िचत् अथात् आम
तव और अिचत् अथात् कृित तव।
वतु तः ये िचत् अथात् आम तव तथा अिचत् अथात् कृित तव ब या ईवर से पृ थक नहीं है बिक ये िविशट
प से ब के ही दो वप ह एवं ब या ईवर पर ही आधािरत ह ।

35 अंतरा टीय सौर गठबंधन (ISA) सौर ऊजा से संपन दे शों का एक संिध आधािरत अंतर-सरकारी संगठन जो सौर ऊजा के
लाभों का दोहन करने और वछ ऊजा अनु योगों को बढ़ावा दे ने के िलए एक वै िवक बाजार णाली बनाने के िलए काय कर
रहा है । इसके बारे म िननिलिखत पर िवचार कीिजए :
1. ISA सदय दे श म मां ग, जोिखम और सं साधन को एक करने और सामं जय बनाने म मदद करता है ।
2. ISA सदय राट  को धन और ौोिगकी दान करता है तथा यापक तर पर सौर अनु योग को िव पोषण, िवकास तथा
उपयोग करने म सहायता करता है ।

नीचे िदए गए कू ट का उपयोग करके सही उर चु िनए :


A. केवल 1
B. केवल 2
C. 1 और 2 दोन
D. कोई नहीं

Your Answer :
Correct Answer : A

Answer Justification :

ISA एक वै िवक सौर बाजार की थापना म कई भूिमकाएं िनभाता है : यह एक उे रक, सम बनाने वाला, एक भं डारक और
एक समवयक के प म काय करता है ।

ISA सदय दे श म मां ग, जोिखम और सं साधन को एक करने और सामं जय बनाने म मदद करता है , इस कार
सदय के िलए लागत कम करने , और नवाचार और िनवे श को उे िरत करने के उे य से एक बड़ा, खरीदार का
बाजार ’बनाता है ।
अं तराट ीय सौर गठबं धन उपयु त द्िवपीय या बहुपीय िवपोषण खोजने म सदय दे श की सहायता करता है ।
ISA पट प से धन या ौोिगकी दान नहीं करता है ; यह उन पिरिथितय को बनाने म मदद करता है जो बड़े
पै माने पर सौर अनु योग को िवपोषण, िवकास और उपयोग करते ह ।

अं तराट ीय सौर गठबं धन सौर ऊजा से सं पन दे श का एक सं िध आधािरत अं तर-सरकारी सं गठन है । ISA की थापना की
पहल भारत के दवारा की गयी थी। इसकी शु आत सं युत प से पे िरस म 30 नवबर, 2015 को सं युत राट जलवायु
समे लन के दौरान COP-21 से पृ थक भारत और ाँस ारा की गई थी।

ISA कक और मकर रे खा के मय आं िशक या पूण प से अविथत 121 सौर सं साधन सं पन दे श का एक अं तराट ीय अं तर-
सरकारी सं गठन है । ISA के मु ख उे य म 1000 गीगावाट से अिधक सौर ऊजा उपादन मता की वै िवक तै नाती और
2030 तक सौर ऊजा म िनवे श के िलये लगभग $1000 िबिलयन की रािश को जु टाना शािमल है । एक िया-उमु ख सं गठन के
प म ISA सौर पिरयोजनाओं को ज़मीनी तर पर ारं भ करने म सहयोग दान करता है ।

https://mnre.gov.in/isa/

https://telegram.me/UpscPdfDrive
https://telegram.me/UPSC_PDF
Total Marks : 200
Online Prelims Full Length Test - 4
( InsightsIAS Mock Test Series for UPSC Preliminary Exam 2020 ) Mark Scored : 0

36 यिद िवशाल जल िनकायों के मय हरे रं ग के धबे दे खे जाते ह , तो यह फ़ाइटोलांकटन की उपिथित के पिरचायक होते है ।
जलीय े  म इस तरह के बड़े धबे के पाए जाने का अथ होगा िक :

A. उस जलीय े  की ाथिमक उपादकता अिधक है ।


B. यह े  ऑसीजन की कमी है ।
C. इस े  का जल असामाय प से गम है ।
D. इसके आस-पास कोई भी तरं ग या धारा नहीं पाई जाती है ।

Your Answer :
Correct Answer : A

Answer Justification :

अिधकां श फाइटोलांकटन ितिनिध नन आं ख से सामायतः नहीं िदखाई दे ते ह । हालां िक, काफी अिधक माा म , कुछ
जाितयां पानी की सतह पर रं गीन धब के प म िदखाई दे सकती ह , जो िक उनकी कोिशकाओं और सहायक रं जक के अं दर
लोरोिफल, जै से िक फाइकोिबिलोिटं स या ज़ थोिफल के कारण होती है ।

फाइटोलांकटन को आमतौर पर माइोएगे कहा जाता है । फाइटोलांकटन या तो काश सं ले षक या केमोसाइने िटक हो
सकते ह । फाइटोलांकटन जलीय खा ृं खला के ाथिमक उपादक म से एक है । फाइटोलांकटन समु दी और ताजे पानी के
आवास दोन म पाया जा सकता है ।

काश सं ले षक फाइटोलांकटन जल िनकाय की ऊपरी धूप की परत म बढ़ता है । रसायन शरीर म फाइटोलांकटन को पानी
के गहरे खं ड म पाया जा सकता है , जहां से कोई भी सूरज की रोशनी नहीं गु जर सकती है ।

फाइटोलांकटन के दो मु ख कार डायटम और डायनोलै गेलेट्स ह । डायटम का आकार एक े , दीघवृ या तारा हो
सकता है । डायटम का महव िसिलका शे ल है , जो डायटम की कोिशका िभि के प म काय करता है ।

डायटम और डाइनोलै गले ट्स की उच वृ दि् ध दर शै वालो के फुटन का कारण बनती है । लाल वार एक कार का समु दी
शै वाल फुिटत है जो बायोटॉिसन का उपादन करता है । ये बायोटॉिसन छोटी मछिलय को नु कसान पहुंचा सकते ह ।

फाइटोलांकटन कई जलीय खा जाल का आधार है । एक सं तुिलत पािरिथितकी तं  म , वे हे ल, झींगा, घघे और
जे िलिफ़श सिहत समु दी जीव की एक िवतृ त ृं खला के िलए भोजन दान करते ह ।

कुछ फाइटोलांकटन बै टीिरया, ोिटट तथा कुछ एकल-कोिशका वाले पादप होते ह । फाइटोलांकटन को अकाबिनक
पोषक तव जै से नाइटे ट् स, फॉफेट और सफर की आवयकता होती है , िजसे वे ोटीन, वसा और काबोहाइडेट म
पिरवितत करते ह ।

37 िननिलिखत कथनों पर िवचार कीिजए :


1. सफोनामाइड्स, हािनकारक बै टीिरया को नट करने वाली सबसे शितशाली दवाओं म से एक है ।
2. िहटामाइन, सूमजीवीय एज ट ारा जिनत शरीर म सूजन को कम करने और िनयं ित करने म मदद करता है ।

उपरोत म से कौन सा सही है / ह 


A. केवल 1
B. केवल 2
C. 1 और 2 दोन
D. कोई नहीं

https://telegram.me/UpscPdfDrive
https://telegram.me/UPSC_PDF
Total Marks : 200
Online Prelims Full Length Test - 4
( InsightsIAS Mock Test Series for UPSC Preliminary Exam 2020 ) Mark Scored : 0

Your Answer :
Correct Answer : D

Answer Justification :

सफ़ोने माइड (Sulfonamide) दय का एक वग िजसके आधार पर अने क दवाएँ बनायी जाती ह ।

मूल जीवाणु रोधी सफोनामाइड िसं थेिटक एं टीमाइोिबयल एज ट होते ह िजनम सफोनामाइड समूह होता है । कुछ
सफोनामाइड्स जीवाणु रोधी गितिविध रिहत होते ह ।

सफोनामाइड्स बै टीिरयोटे िटक ह और बै टीिरया के िवकास और गु णन को रोकते ह , ले िकन उह नहीं मारते ह ।

िहटामाइन एक काबिनक नाइट ोजनयु त यौिगक है जो थानीय ितरा ितियाओं म शािमल है , साथ ही आं त म
शारीिरक िया को िविनयिमत करने और मितक, रीढ़ की हड्डी और गभाशय के िलए एक यूरोट ास
ं मीटर के प म काय
करता है । िहटामाइन उे रक ितिया म शािमल होता है ।

एं टी-िहटामाइन यौिगक की कारवाई को रोकते ह , िहटामाइन जो शरीर म सूजन का कारण बनता है । ऐसे कई तरीके ह
िजनसे िहटामाइन की िया को अव िकया जा सकता है ।

38 सरकार ने दे श म सौर ऊजा उपादन बढ़ाने के िलए िननिलिखत म से कौन सा/से कदम उठाये है ?
1. अं तर राय पारे षण णाली (ISTS) की छटू और कुछ पिरयोजनाओं के िलए सौर ऊजा की अं तर-रायीय िबी के िलए
शु क और नु कसान।
2. वचािलत माग के तहत 100 ितशत तक िवदे शी य िनवे श (FDI) की अनु मित।
3. नवीकरणीय ऊजा मता सं वधन के िग ड एकीकरण की सु िवधा के िलए हिरत ऊजा गिलयारे पिरयोजना का कायावयन।
4. सौर शहर का िवकास।

नीचे िदए गए कू ट का उपयोग करके सही उर चु िनए :


A. केवल 2
B. केवल 1, 3 और 4
C. केवल 2 और 3
D. 1, 2, 3 और 4

Your Answer :
Correct Answer : D

Answer Justification :

सरकार ने दे श म सौर ऊजा उपादन बढ़ाने के िलए िननिलिखत कदम उठाए ह :

िदसं बर, 2022 तक 100 गीगावॉट सौर ऊजा मता थािपत करने के लय की घोषणा।
अं तर राय पारे षण णाली (ISTS) की छट
ू ।
वचािलत माग के तहत 100 ितशत तक िवदे शी य िनवे श (एफडीआई) की अनु मित।
लाइस सधािरय को लागत भावी और पारदशी तरीके से ितपधी दर पर सौर और पवन ऊजा की खरीद करने तथा
िवतरण करने म सम बनाने के िलए मानक िनिवदा आमं ण िदशािनद श की अिधसूचना।
बड़े पै माने पर नवीकरणीय ऊजा मता बढ़ाने के िग ड एकीकरण की सु िवधा के िलए हिरत ऊजा गिलयारे पिरयोजना
का ियावयन।
सौर फोटोवोिटक णाली / उपकरण के उपयोग के िलए गु णवा मानक की अिधसूचना।

https://telegram.me/UpscPdfDrive
https://telegram.me/UPSC_PDF
Total Marks : 200
Online Prelims Full Length Test - 4
( InsightsIAS Mock Test Series for UPSC Preliminary Exam 2020 ) Mark Scored : 0

धान मं ी िकसान उजा सु रा योजना उथान महािभयान योजना (PM-KUSUM), CPSU (सरकारी उपादक)
योजना-द्िवतीय चरण और सौर छत चरण II कायम सिहत िविभन योजनाओं का शु भारं भ।

नई और नवीकरणीय ऊजा मं ालय ने अपनी योजना "सोलर िसटीज का िवकास" के तहत 13 पायलट और 5 मॉडल शहर
सिहत 60 शहर को 12 वीं पं चवषीय योजना अविध म मं जरू ी / मं जरू ी दी है ।

तािवत 60 सौर शहर म से , 50 शहर के िलए मं जरू ी दे दी गयी ह , िजनम नई िदली, आगरा, चं डीगढ़, गु ड़गां व,
फरीदाबाद, अमृ तसर, यू टाउन (कोलकाता), हावड़ा, मयग ाम, कोिच और भोपाल शािमल ह ।

https://www.thehindu.com/news/national/govt-approval-to-develop-50-solar-cities/article7571826.ec
e

39 ‘िवव रोज़गार और सामािजक दृिटकोण झान िरपोट ’ (WESO) का काशन िकसके ारा िकया गया है 

A. िवव आिथक मं च (WEF)


B. अं तराट ीय म सं गठन (ILO)
C. िवव ब क (WB)
D. अं तराट ीय मु दा कोष (IMF)

Your Answer :
Correct Answer : B

Answer Justification :

सं युत राट अं तराट ीय म सं गठन (ILO) ने िवव रोज़गार और सामािजक दृिटकोण झान िरपोट (WESO Trends
Report), 2020 को कािशत िकया है ।

वािषक WESO ट ड िरपोट म मु ख म बाज़ार के मु  का िवले षण िकया गया है , िजसम बे रोज़गारी, म का अभाव,
कायशील गरीबी, आय असमानता, म-आय िहसे दारी आिद कारक लोग को उनकी ितभा के अनु प रोज़गार से दरू करते
ह ।

आिथक िवकास को दे खते हुए यह पता चलता है िक िवकास की वतमान गित और वप गरीबी को कम करने एवं कम आय
वाले दे श म काम करने की िथित म सु धार के यास म सबसे बड़ी बाधा ह ।

40 यूरोिपयन मु त यापार संघ (EFTA) एक अंतर सरकारी संगठन है , िननिलिखत म से कौन से दे श EFTA म शािमल है ?
1. आइसल ड
2. नॉव
3. जमनी
4. ांस

नीचे िदए गए कू ट का उपयोग करके सही उर चु िनए :


A. केवल 1 और 2
B. केवल 3 और 4
C. केवल 2, 3 और 4
D. केवल 1 और 3

https://telegram.me/UpscPdfDrive
https://telegram.me/UPSC_PDF
Total Marks : 200
Online Prelims Full Length Test - 4
( InsightsIAS Mock Test Series for UPSC Preliminary Exam 2020 ) Mark Scored : 0

Your Answer :
Correct Answer : A

Answer Justification :

EFTA एक े ीय यापार सं गठन और मु त यापार े  है िजसम चार यूरोपीय दे श शािमल ह : आइसल ड, लीकट टीन
(Liechtenstein), नॉव और िवट् जरल ड।

EFTA की थापना उन यूरोपीय राय के िलए एक वै किपक यापार लॉक के प म की गई थी जो तकालीन


यूरोपीय आिथक समु दाय (EEC) म शािमल होने म असमथ या अिनछुक थे (जो बाद म यूरोपीय सं घ के दय बन
गए)।
यह सं गठन, यूरोिपयन एकल बाज़ार म इसके चार मु य सदय तथा यूरोिपयन सं घ (EU) की सहभािगता के
समानांतर काय करता है ।
EFTA की कोई राजनीितक शाखा नहीं है , न ही यह कोई कानून पािरत करता है और न ही िकसी कार के सं घ की
थापना करता है ।

41 िननिलिखत कथनों पर िवचार कीिजए :


1. IRIX, भारतीय और वै िवक समु दाय के बीच ऊजा के सं रण के िलए िवचार के आदान-दान को ोसािहत करने के िलए
एक मं च है ।
2. IRIX का आरभनीित आयोग ारा िकया गया था।

नीचे िदए गए कू ट का उपयोग करके सही उर चु िनए :


A. केवल 1
B. केवल 2
C. 1 और 2 दोन
D. उपरोत म से कोई नहीं

Your Answer :
Correct Answer : A

Answer Justification :

नवीन और नवीकरणीय ऊजा मं ालय (MNRE) ारा शु  और सं चािलत, IRIX एक ऐसा मं च है जो ऊजा के ित जागक
भारतीय और वै िवक समु दाय के बीच िवचार के आदान-दान को बढ़ावा दे ता है ।

आईआरआईएस के बारे म :

यह "ऊजा जागक यितय का एक चु िनं दा समूह है , जो भारत म नई और नवीकरणीय ऊजा के िवकास और उपभोग
की सु िवधा दान करने के हे तु िवचार का आदान-दान करते ह तथा काययोजना बनाते ह ।
यिद आपके पास सोलर एनजी, िवं ड एनजी, बायोमास, हाइडो, वे ट टू एनजी, हाइडोजन एनजी और अय िरयूएबल
एनजी िरसोस ज म म टरिशप या आइिडया की तलाश है , तो यह आपके िलए ले टफॉम है ।
उोगपितय तथा िवशे ष के िलए, यह ले टफ़ॉम ान-साझे दारी और सं भािवत प से भारत की कुछ सबसे बड़ी
िचं ताओं का हल खोजने के िलए एक वे श ार है ।

http://irix.gov.in/about_the_program

42 उर-पूव मानसून या वापस लौटते हुए मानसून के सदभ म िननिलिखत कथनों पर िवचार कीिजए :

https://telegram.me/UpscPdfDrive
https://telegram.me/UPSC_PDF
Total Marks : 200
Online Prelims Full Length Test - 4
( InsightsIAS Mock Test Series for UPSC Preliminary Exam 2020 ) Mark Scored : 0

1. मानसून के मौसम के बाद सूय दिण की ओर हटने लगता है ।


2. भारतीय उपमहाीप का उरी भू-भाग शु क गमी के बाद ठं डा होने लगता है ।
3. यह दे श के दिणी िहसे म पूणतया शु क मौसम होता है ।

नीचे िदए गए कू ट का उपयोग करके सही उर चु िनए :


A. केवल 1 और 2
B. केवल 2 और 3
C. केवल 1 और 3
D. 1, 2 और 3

Your Answer :
Correct Answer : A

Answer Justification :

पूवोर मानसून के मौसम म दिणी ायीप भाग म तटीय आं ध दे श, रायलसीमा और तिमलनाडु-पां िडचे री से सवािधक
वषा होती है । तिमलनाडु म वािषक वषा का लगभग 48% वषा उर-पूव मानसून या वापस लौटते हुए मानसून के मौसम म
होती है ।

िसतं बर के आसपास, सूरज ते जी से दिण की ओर हटने लगता है , िजससे भारतीय उपमहाीप का उरी भूभाग ते जी
से ठं डा होने लगता है ।
इसके साथ ही उरी भारत पर वायु दाब म वृ दि् ध होने लगती है ।
इस कारण िहमालय तथा गं गा के मै दान से हवाएं दिणी ायीप की ओर वािहत होने लगती है तथा इसे पूवोर
मानसून या िरट ीिटं ग मानसून के प म जाना जाता है ।

43 िननिलिखत कथनों पर िवचार कीिजए :


1. अं तराट ीय नवीकरणीय ऊजा एज सी (IRENA) एक अं तरसरकारी सं गठन है जो नवीकरणीय ऊजा सं बंिधत नीितगत मामल
म उकृटता कद तथा ान का भं डार है ।
2. भारत IRENA के सं थापक सदय म से एक है ।

नीचे िदए गए कू ट का उपयोग करके सही उर चु िनए :


A. केवल 1
B. केवल 2
C. 1 और 2 दोन
D. कोई नहीं

Your Answer :
Correct Answer : C

Answer Justification :

भारत अं तराट ीय नवीकरणीय ऊजा एज सी (IRENA) के सं थापक सदय म से एक है । IRENA एक अं तरसरकारी सं गठन
है जो एक थायी ऊजा भिवय के िलए सहयोग और नवीकरणीय ऊजा को अपनाने और थायी उपयोग को बढ़ावा दे ता है ।

IRENA की थापना 2009 म हुई थी और यह 8 जु लाई 2010 से भाव म आया तथा इसका मु यालय मसदर िसटी,
अबू धाबी म है । यह सं युत राट का एक आिधकािरक पयवेक है ।
अं तराट ीय नवीकरणीय ऊजा एज सी (IRENA) नवीकरणीय ऊजा सं बंिधत नीितगत मामल म उकृटता कद तथा

https://telegram.me/UpscPdfDrive
https://telegram.me/UPSC_PDF
Total Marks : 200
Online Prelims Full Length Test - 4
( InsightsIAS Mock Test Series for UPSC Preliminary Exam 2020 ) Mark Scored : 0

ान का भं डार है ।
IRENA सतत िवकास, ऊजा पहुंच, ऊजा सु रा और िनन-काबन आिथक िवकास और समृ दि् ध हे तु जै व ऊजा,
भूतापीय, जल िवु त, महासागर, सौर और पवन ऊजा सिहत नवीकरणीय ऊजा के सभी प को अपनाने और थायी
उपयोग को बढ़ावा दे ता है ।

https://mnre.gov.in/international-relations/international-organisations

44 िननिलिखत कथनों पर िवचार कीिजए :


1. औषिध (मूय िनयं ण) आदे श (DPCO) आवयक वतु अिधिनयम (ECA), 1955 के तहत जारी िकया गया एक आदे श है ।
2. DPCO, मूय िनयं ित दवाओं की सूची, दवाओं के मूयो को िनधािरत करने की िया का िनधारण करता है ।
3. राट ीय औषध मूय िनधारण ािधकरण (NPPA) औषध उपाद के मूय के िनधारण / सं शोधन तथा DPCO के ावधान
को लागू करने के िलए िजमे दार है ।

नीचे िदए गए कू ट का उपयोग करके सही उर चु िनए :


A. केवल 1
B. केवल 2 और 3
C. केवल 1 और 3
D. 1, 2 और 3

Your Answer :
Correct Answer : D

Answer Justification :

औषिध (मूय िनयं ण) आदे श (DPCO) आवयक वतु ए अिधिनयम 1955 की धारा 3 के अतगत भारत सरकारा ारा
दवाओं के मूय को यविथत करने के िलए जारी िकया गया एक आदे श है । यह आदे श अय बात के साथ-साथ मूय
िनयं ित दवाओं की सूची, दवाओं के मूयो को िनधािरत करने की िया, सरकार ारा िनधािरत िकए गए मूय के
कायावयन की िविध, ावधान के उलं घन के िलए दं ड इयािद दान करता ह ।

DPCO के ावधान को कायोिवत करने के उे य से सरकार की शितयाँ राट ीय औषध मूय िनधारण ािधकरण (NPPA)
म िनिहत है ।

राट ीय औषध मूय िनधारण ािधकरण की थाना 29 अगत 1997 को, मं िमं डल सिमित के ारा दवा नीित का िसतबर
1994 म पु नवलोकन करते समय िलए गए एक िनणय के अनु सार, िवशे ष के एक वतं  िनकाय के प म की गई थी।

ािधकरण को अय कायों के साथ-साथ औषध उपाद (पु ं ज दवाओं और िविनिमती) के मूय के िनधारण / सं शोधन का
काय, दवा (मूय िनयं ण) आदे श के ावधान को लागू करना एवं दे श म िनयं ित और अिनयं ित (सरकारी िनयं ण रिहत)
दवाओं के मूय की िनगरानी के काय सौप गए ह ।

45 महामा गांधी राटीय ग ामीण रोजगार गारंटी योजना (MGNREGS) के लय या ह ?
1. पं चायती राज सं थाओं को मजबूत बनाना।
2. ग ामीण े  के ाकृितक सं साधन आधार का कायाकप करना।
3. थाई और उपादक ग ामीण सं पि का आधार िनमाण।

नीचे िदए गए कू ट का उपयोग करके सही उर चु िनए :


A. केवल 1 और 3

https://telegram.me/UpscPdfDrive
https://telegram.me/UPSC_PDF
Total Marks : 200
Online Prelims Full Length Test - 4
( InsightsIAS Mock Test Series for UPSC Preliminary Exam 2020 ) Mark Scored : 0

B. केवल 3
C. 1, 2 और 3
D. केवल 1

Your Answer :
Correct Answer : C

Answer Justification :

औिचय: MGNREGS के मु य उे य ह :

ये क पिरवार के अकुशल वयक सदय (18 वष और अिधक) को हर िवीय वष म कम से कम सौ िदन का रोजगार
दे ना है ।
इससे ग ामीण इलाक म पिरवार की आजीिवका बढे़ गी व थायी, सामु दाियक, सामािजक और आिथक पिरसपिय
का िवकास होगा।
गरीब के आजीिवका सं साधन आधार को मजबूत करना ;
सामािजक समावे श को सु िनिचत करना और
पं चायती राज सं थाओं को मजबूत बनाना।

MGNREGA के लय या ह 

1. रोजगार के अवसर की गारं टी दे कर ग ामीण भारत म रहने वाले सबसे कमजोर लोग के िलए सामािजक सु रा।
2. थायी पिरसं पिय के िनमाण हे तु कायों म मजदरू ी रोजगार के अवसर के िनमाण के मायम से ग ामीण गरीब की
आजीिवका सु रा बढ़ाना।
3. ग ामीण े  के ाकृितक सं साधन आधार का कायाकप।
4. एक थाई और उपादक ग ामीण सं पि का आधार िनमाण।
5. अिधकार-आधािरत कानून की ियाओं के मायम से सामािजक प से वं िचत, िवशे ष प से मिहलाओ,ं अनु सिू चत
जाितय (एससी) और अनु सिू चत जनजाितय (एसटी) का सशितकरण।
6. िविभन गरीबी और आजीिवका पहल के अिभसरण के मायम से िवकेदीकृत, सहभागी योजना को सु दृढ़ बनाना।
7. पं चायती राज सं थाओं को मजबूत करके जमीनी तर पर लोकतं  को गहरा करना।

https://nrega.nic.in/Circular_Archive/archive/nrega_doc_FAQs.pdf

46 इटली की सीमा िननिलिखत म से िकन दे शों के साथ िमलती है 


1. जमनी
2. िवट् जरल ड
3. ऑिट या
4. हं गरी

नीचे िदए गए कू ट का उपयोग करके सही उर चु िनए :


A. केवल 1 और 2
B. केवल 2, 3 और 4
C. केवल 2 और 3
D. केवल 1, 3 और 4

Your Answer :

https://telegram.me/UpscPdfDrive
https://telegram.me/UPSC_PDF
Total Marks : 200
Online Prelims Full Length Test - 4
( InsightsIAS Mock Test Series for UPSC Preliminary Exam 2020 ) Mark Scored : 0

Correct Answer : C

Answer Justification :

औिचय :

47 शहरी, औोिगक और कृिष अपिशट / अवशे षों से ऊजा पर काय म िदशािनद श’ के संदभ म िननिलिखत कथनों पर िवचार
कीिजए :
1. इस योजना म औोिगक अपिशट पिरयोजनाओं से बायोगै स उपादन हे तु पूंजीगत सिसडी तथा अनु दान सहायता के प
म कदीय िवीय सहायता दान की जाती है ।
2. बायोमै थेनेशन (Biomethanation) तथा और पायरोिलिसस ौोिगकी से सं बंिधत पिरयोजनाओं को कायम के अं तगत
योय तकनीक माना गया है ।

नीचे िदए गए कू ट का उपयोग करके सही उर चु िनए :


A. केवल 1
B. केवल 2
C. 1 और 2 दोन
D. उपरोत म से कोई नहीं

Your Answer :
Correct Answer : C

Answer Justification :

शहरी, औोिगक और कृिष अपिशट / अवशे ष से ऊजा पर कायम िदशािनद श’ (2017-18, 2018-19 और 2019-20) के
िलए पिरणाम

यह योजना िननिलिखत गितिविधय के सं बंध म पूंजीगत सिसडी और अनु दान सहायता के प म कदीय िवीय सहायता
दान करती है :

https://telegram.me/UpscPdfDrive
https://telegram.me/UPSC_PDF
Total Marks : 200
Online Prelims Full Length Test - 4
( InsightsIAS Mock Test Series for UPSC Preliminary Exam 2020 ) Mark Scored : 0

बायोमै थेनेशन के मायम से औोिगक अपिशट, सीवे ज ट ीटम ट लांट, शहरी और कृिष अपिशट / अवशे ष से
बायोगै स उपादन।
औोिगक अपिशट, सीवे ज उपचार सं यं, शहरी और कृिष अपिशट / अवशे ष से उपािदत बायोगै स से जै व-
सीएनजी / समृ  बायोगै स की िबजली उपादन या उपादन।
उोग म बायोमास गै सीफायर आधािरत पिरयोजनाओं की थापना, िबजली उपादन के िलए कैिटव पावर और
चावल िमल और अय उोग और गां व म काश, जल पिपं ग और सूम उम की तापीय आवयकताओं की
पूित के िलए।
अनु संधान और िवकास, सं साधन मूयांकन, ौोिगकी उनयन और दशन मूयांकन आिद सिहत चार
गितिविधयाँ

कायम के मु य उे य इस कार ह :

बायोगै स / BioCNG / समृ  बायोगै स / शहरी, औोिगक और कृिष कचरे से िबजली के प म ऊजा की वसूली के िलए
पिरयोजनाओं की थापना को बढ़ावा दे ने के िलए ; और उोग म गै सीकरण के मायम से कैिटव पावर और थमल उपयोग।

बायोमास गै सीफायर आधािरत िबजली सं यं को बढ़ावा दे ने के िलए कैिटव पावर और राइस िमल और अय उोग और
गां व की काश, जल पिपं ग और सूम उम की ताकािलक जरत को पूरा करने के िलए िबजली उपादन।

https://mnre.gov.in/img/documents/uploads/68e223e9afdb4c5281413de6dc2174d4.pdf

48 ‘भारतमाला पिरयोजना’ के संदभ म िननिलिखत कथनों पर िवचार कीिजए :


1. यह राजमाग िवकास के िलए एक अबेला कायम है जो दे श म सड़क यातायात की मता सु धार पर किदत है ।
2. यह पिरयोजना पूरी तरह से िनजी ोत, घरे लू और िवदे शी दोन, ारा िव पोिषत है तथा इस पिरयोजना की दे खरे ख
सरकारी एज िसय ारा की जा रही है ।
3. इसके अं तगत आिथक कॉरीडोर, फीडर कॉरीडोर और इं टर कॉरीडोर, राट ीय कॉरीडोर, तटवती सड़क, बं दरगाह सं पक सड़क
आिद का िनमाण िकया जाएगा।

नीचे िदए गए कू ट का उपयोग करके सही उर चु िनए :


A. केवल 1 और 2
B. केवल 3
C. केवल 1 और 3
D. 1, 2 और 3

Your Answer :
Correct Answer : C

Answer Justification :

भारतमाला राट ीय राजमाग िवकास पिरयोजना है । यह राजमाग िवकास के िलए एक अबेला कायम है जो दे श म सड़क
यातायात की मता सु धार पर किदत है । इसके तहत नए राजमागों के िनमाण के अलावा उन पिरयोजनाओं को भी पूरा िकया
जाएगा जो अब तक अधूरी ह । इस पिरयोजना म सीमावती और ‘इं टरने शनल कने िटिवटी’ वाली िवकासामक पिरयोजनाएँ ,
बं दरगाह को सड़क से जोड़ना, ने शनल कॉिरडोस को और बे हतर बनाना भी शािमल है ।

इस पिरयोजना के तहत िनमाण काय करने वाली मु य एज िसयाँ , भारतीय राट ीय राजमाग, राट ीय राजमाग और औोिगक
िवकास िनगम तथा लोक िनमाण िवभाग ह ।

यह महवपूण बु िनयादी ढाँचे के अं तराल को कम करके करे गा। पिरयोजना म राट ीय राजमागों के साथ फीडर मागों का

https://telegram.me/UpscPdfDrive
https://telegram.me/UPSC_PDF
Total Marks : 200
Online Prelims Full Length Test - 4
( InsightsIAS Mock Test Series for UPSC Preliminary Exam 2020 ) Mark Scored : 0

िनमाण शािमल है ।

भारतमाला पिरयोजना का लगभग 80% सरकारी िव पोिषत, इं जीिनयिरं ग खरीद और िनमाण (EPC) मॉडल पर आधािरत
होगा, जबिक बाकी एक हाइिबड-वािषकी सावजिनक िनजी भागीदारी होगी।

भारतमाला पिरयोजना म कम से कम 5 लाख करोड़ पये की लागत से दे श भर म 44 आिथक गिलयार की कपना की गयी
है । इसके अं तगत आिथक कॉरीडोर, फीडर कॉरीडोर और इं टर कॉरीडोर, राट ीय कॉरीडोर, तटवती सड़क, बं दरगाह सं पक
सड़क आिद का िनमाण िकया जाएगा।

49 ‘नीित-िनमा णक’ गितिविधयाँ िननिलिखत म से िकस े  म से वा े  के अंतग त आती ह ?

A. तृ तीयक गितिविधयाँ
B. पं चक गितिविधय
C. चतु थक गितिविधयाँ
D. उपरोत सभी

Your Answer :
Correct Answer : B

Answer Justification :

यह NCERT की एक तािलका है जो आपको िविभन से वा कार के बीच के सूम भे द को याद रखने म मदद करे गी।

50 िननिलिखत कथनों पर िवचार कीिजए :


1. वतमान म , भारत केवल ने पाल और भूटान को िबजली का िनयात करता है तथा बां लादे श और यांमार से िबजली आयात
करता है ।
2. ने पाल और बां लादे श दोन दे श के बीच ऊजा यापार करने के िलए भारतीय ट ास
ं िमशन िग ड का उपयोग करते ह ।

नीचे िदए गए कू ट का उपयोग करके सही उर चु िनए :


A. केवल 1
B. केवल 2
C. 1 और 2 दोन
D. उपरोत म से कोई नहीं

Your Answer :
Correct Answer : D

https://telegram.me/UpscPdfDrive
https://telegram.me/UPSC_PDF
Total Marks : 200
Online Prelims Full Length Test - 4
( InsightsIAS Mock Test Series for UPSC Preliminary Exam 2020 ) Mark Scored : 0

Answer Justification :

वतमान म , भारत ने पाल, बां लादे श और यांमार को िबजली िनयात करता है , जबिक भारत भूटान से िबजली आयात करता
है । हालां िक, कभी-कभी भारत कम वषा वाले मौसम म भूटान को िबजली िनयात करता है ।

ने पाल और बां लादे श दो दे श के बीच यापार करने के िलए भारतीय ट ास


ं िमशन िग ड का उपयोग नहीं कर रहे ह ।

ने पाल, भूटान और बां लादे श सिहत पड़ोसी दे श के साथ िबजली के आयात / िनयात के िलए यापार यवथा, े ीय
यापार को सु गम बनाएगी और सं बंिधत दे श म िबजली की आवयकता को पूरा करने म मदद करे गी िजससे े  म अिधक से
अिधक ऊजा सु रा हो सके।

भारत म पहले से ही उच वोटे ज िसं ोनस (अटरने िटं ग करं ट) और एिसं ोनस (हाई वोटे ज डायरे ट करं ट) कने शन के
मायम से बां लादे श, भूटान और ने पाल के साथ े ीय िबजली यवथा समिवत है । उनत राट  म चिलत तकनीको
जै से STATCOM, वोटे ज सोस कवट र आधािरत HVDC णाली, आिद को े ीय पड़ोसी दे श म िववसनीयता के साथ
िबजली हतांतरण की सु िवधा के िलए सु धार के सतत उपाय के प म भारतीय िग ड म योग िकया गया है ।

https://pib.nic.in/newsite/pmreleases.aspx?mincode=52 (Release ID :200468)

51 संिवधान के अंतग त संसदीय सिमितयां अपनी शितयां िननिलिखत म से िकससे संबंिधत अनु छे दों से ात करती ह 
1. सं सद सदय के िवशे षािधकार
2. सं सदीय सिमितय की िया तथा काय सं चालन के िनयम को िविनयिमत करने हे तु सं सद का अिधकार

नीचे िदए गए कू ट का उपयोग करके सही उर चु िनए :


A. केवल 1
B. केवल 2
C. 1 और 2 दोन
D. उपरोत म से कोई नहीं

Your Answer :
Correct Answer : C

Answer Justification :

सं सदीय सिमितयां अनु छे द 105 (सं सद सदय के िवशे षािधकार) और अनु छे द 118 (सं सदीय सिमितय की िया तथा
काय सं चालन के िनयम को िविनयिमत करने हे तु सं सद का अिधकार) से अपना अिधकार ात करती ह ।

कृित के आधार पर सं सदीय सिमितयाँ दो कार की होती ह : थायी सिमितयां तथा अथायी या तदथ सिमितयां

थायी सिमित : ये थायी एवं िनयिमत सिमित होती है , िजसका गठन सं सद के अिधिनयम के उपबं ध अथवा लोकसभा के
काय-सं चालन िनयम के अनु सरण म िकया जाता है ।

इसम िननिलिखत सिमितयाँ शािमल ह : लोक ले खा सिमित, ाकलन सिमित, सावजिनक उपम सिमित, अनु सिू चत जाित
व अनु सिू चत जनजाित समु दाय के कयाण सं बंधी सिमित, कायमंणा सिमित, िवशे षािधकार सिमित, िवभागीय सिमित।

अथायी या तदथ सिमित : योजन िवशे ष के िलये तदथ सिमित का िनमाण िकया जाता है और काय पूरा होने के पचात्
इसका अितव समात हो जाता है ।

यह भी दो कार की होती ह : जाँच सिमितयाँ तथा सलाहकार सिमितयाँ

https://telegram.me/UpscPdfDrive
https://telegram.me/UPSC_PDF
Total Marks : 200
Online Prelims Full Length Test - 4
( InsightsIAS Mock Test Series for UPSC Preliminary Exam 2020 ) Mark Scored : 0

जाँच सिमितयाँ : िकसी ताकािलक घटना की जाँच के िलये ।

सलाहकार सिमितयाँ : िकसी िवधे यक इयािद पर िवचार करने के िलये ।

https://www.insightsonindia.com/2020/03/25/insights-daily-current-affairs-pib-summary-25-march-2
020/

52 समाचारों म चिच त ‘सु पर ििटकल और अटा-सु पर ििटकल (USC) लांट टे नोलॉजी’ िननिलिखत म िकससे संबंिधत है 

A. नै रोबी पै केज’ के अं तगत तहत यापार शत


B. TRIPS के तहत बीडर के अिधकार
C. वायु मं डलीय दष ू ण म कमी
D. यूजन िरएटर ौोिगकी

Your Answer :
Correct Answer : C

Answer Justification :

योजना आयोग ारा जारी 2006 की एकीकृत ऊजा नीित िरपोट म सु परििटकल ौोिगकी की अवधारणा को एक राट ीय
योजना के प म पे श िकया गया था।

यह नोट िकया गया िक भारतीय िबजली सं यं की औसत धन पांतरण दता िसफ 30.5 ितशत है , हालां िक नए
500 मे गावाट के सं यं की दता 36 ितशत है ।
अयाधु िनक सु परििटकल बॉयलर सं यं के थान के आधार पर 46 ितशत की दता तर तक पहुंच सकते ह ।
अिधक मता होने के साथ-साथ, इन िरएटर म CO2 उसजन को कम करने और PM उसजन को लगभग समात
करने की मता भी है ।
कोयला िबजली सं यं से उसजन को कम करने के बारे म जलवायु पिरवतन पर पे िरस समझौते के कुछ लय को
ात िकया जाएगा।

53 ‘िज़यारत’ शद बहुधा सूिफयों के संदभ म सुना जाता है , िननिलिखत म यह िकससे संबंिधत है 

A. िदय नाम का पाठ


B. यािय को आय और भोजन दे ना
C. मु ख धािमक कद की तीथयाा
D. गु  और िशय के बीच सं वाद

Your Answer :
Correct Answer : C

Answer Justification :

िज़यारत अरबी शद है : िजस का अथ "याा करना" या उस थल का "दशन" करना।

इलाम म यह पिव याा, पिव थान, क़ब या तीथथल की तीथ याा को सं दिभत करता है । ईरानी और दिण एिशयाई
मु सलमान हज याा के िलए िज़यारत शद का इते माल करते ह और साथ ही तीथयािय के िलए अय थान जै से पिव
थान पर जाने को भी िज़यारत शद ही उपयोग करते ह । जै से दरगाह की िज़यारत करना, क़ब की िज़यारत करना, क़िबतान

https://telegram.me/UpscPdfDrive
https://telegram.me/UPSC_PDF
Total Marks : 200
Online Prelims Full Length Test - 4
( InsightsIAS Mock Test Series for UPSC Preliminary Exam 2020 ) Mark Scored : 0

की िज़यारत करना।

यह पै गबर मु हमद व उनके पिरवार के सदय और वं शज से जु ड़े थल की तीथ याा का एक प है । िस सूफी सं त
की कब के पास सूफी और पूजा थल बनाए जाते ह ; उह असर खानकाह कहा जाता है ।

54 भारत के दिण-पूवी तटों पर िननिलिखत म से िकस कार के वन पाए जाते ह ?

A. उणकिटबं धीय पणपाती वन


B. उणकिटबं धीय वषावन
C. दोन (a) और (b)
D. उपरोत म से कोई नहीं

Your Answer :
Correct Answer : C

Answer Justification :

वषावन पिचमी घाट (दिणी-पिचमी तट के पास) और लीप, अं डमान और िनकोबार, असम के ऊपरी िहस और
तिमलनाडु तट (दिण-पूवी तट) के ीप समूह म भारी वषा वाले े  तक सीिमत ह ।

वे शु क शु क मौसम के साथ 200 से मी से अिधक वषा वाले े  इनके िलए अनु कूल होता है । ये वृ  बहुत ऊँचे होते ह ।

पणपाती वन के िलए, दिण-पूवी वषा को दे खते हुए, मु य प से दिण-पूवी तट के अितिरत दिणी ायीपीय भारत
म पाए जाते ह ।

https://telegram.me/UpscPdfDrive
https://telegram.me/UPSC_PDF
Total Marks : 200
Online Prelims Full Length Test - 4
( InsightsIAS Mock Test Series for UPSC Preliminary Exam 2020 ) Mark Scored : 0

55 ‘िभलसा टॉस’ (Bhilsa Topes) के बारे म िननिलिखत पर िवचार कीिजए :


1. यह जे स मु लर ारा वातु कला के साय के आधार पर बौ धम के इितहास को िनधािरत करने का यास था।
2. यह ‘सांची’ के बारे म िलिखत आरं िभक िववरण म से एक था।

उपरोत म से कौन सा सही है / ह 


A. केवल 1
B. केवल 2
C. 1 और 2 दोन
D. उपरोत म से कोई नहीं

Your Answer :
Correct Answer : B

Answer Justification :

पु रातव िवभाग के उच पद पर रहते हुए किनं घम ने भारत के ाचीन िवमृ त इितहास के िवषय म काफी जानकारी सं सार के
सामने रखी। ाचीन थान की खोज और अिभले ख एवं िसक के सं गहण ारा उहने भारतीय अतीत के इितहास की शोध
के िलए मूयवान् सामग ी जु टाई और िवान के लए इस िदशा म काय करने का माग शत कर िदया।

1854 म उहने ‘द िभसा टॉस, वातु कला माण के आधार पर बौ धम के इितहास को थािपत करने का यास’
कािशत िकया।

https://telegram.me/UpscPdfDrive
https://telegram.me/UPSC_PDF
Total Marks : 200
Online Prelims Full Length Test - 4
( InsightsIAS Mock Test Series for UPSC Preliminary Exam 2020 ) Mark Scored : 0

56 आिथ क मामलों के िवभाग (DEA) के कायो के बारे म िननिलिखत पर िवचार कीिजए :


1. यह भु गतान सं तुलन सिहत िवदे शी िनवे श और िवदे शी मु दा सं साधन की िनगरानी करता है ।
2. यह कदीय बजट की तै यारी के िलए िजमे दार है ।
3. यह सरकार के िलए कराधान और बाजार ऋण के मायम से आं तिरक सं साधन जु टाने के िलए िजमे दार है ।

नीचे िदए गए कू ट का उपयोग करके सही उर चु िनए :


A. केवल 1
B. केवल 2 और 3
C. केवल 2
D. 1, 2 और 3

Your Answer :
Correct Answer : D

Answer Justification :

आिथक मामल के िवभाग (DEA) वतमान म आिथक झान पर सरकार को आिथक बं धन के आं तिरक और बा सभी
पहलु ओं पर सलाह दे ती है िजसम मूय, ऋण, राजकोषीय और मौिदक नीित और िनवे श िनयम शािमल ह ।

साथ ही यह िवभाग भारत सरकार टकसाल, मु दा े स, से यु िरटी े सेसड से यु िरटी पे पर िमस के बं धन के अलावा
राट ीयकृत ब क, जीवन और जनरल इं योर स से सं बंिधत नीितय की पयवेण करते ह ।
भारत ारा ात सभी बाहरी िवीय और तकनीकी सहायता, एफएओ, आईएलओ, यूिनडो जै से िवशे ष अं तराट ीय
सं गठन के मायम को छोड़कर और िवान और ौोिगकी, सं कृित और िशा के े  म अं तरराट ीय/द्िवपीय
िवशे ष समझौते के अधीन को छोड़कर इस िवभाग ारा िनगरानी की जाती है ।
साथ ही आिथक मामल के िवभाग (DEA) राट पित िनयम और सं घीय े  शासन के तहत केदीय बजट और
राय सरकार के बजट को सं सद म तु त करने के िलए भी िजमे दार है ।
यह दे श की आिथक नीितय और आिथक बं धन के घरे लू और अं तराट ीय पहलु ओं पर असर डालने वाले कायम
को तै यार करने और िनगरानी करने के िलए कद सरकार की नोडल एज सी है । इस िवभाग की एक मु ख िजमे दारी
कदीय बजट को सालाना (रे लवे बजट को छोड़कर) तै यार करना है ।

57 'उपयोिगता पे टट' (Utility patents)के बारे म िननिलिखत पर िवचार कीिजए :


1. ये मामूली सु धारो अथवा दता म सु धार के िलए िदए गए पे टट ह जो पे टट नवाचार के अनु प नहीं ह ।
2. भारत उपयोिगता पे टट सु रा दान नहीं करता है ।

उपरोत म से कौन सा सही है / ह 


A. केवल 1
B. केवल 2
C. 1 और 2 दोन
D. उपरोत म से कोई नहीं

Your Answer :
Correct Answer : C

Answer Justification :

'उपयोिगता पे टट', मामूली सु धारो अथवा दता म सु धार के िलए िदए गए पे टट ह जो पे टट नवाचार के अनु प नहीं होते
ह ।

https://telegram.me/UpscPdfDrive
https://telegram.me/UPSC_PDF
Total Marks : 200
Online Prelims Full Length Test - 4
( InsightsIAS Mock Test Series for UPSC Preliminary Exam 2020 ) Mark Scored : 0

सभी दे श उपयोिगता पे टट सु रा दान नहीं करते ह । भारत उनम से एक है । भारत म उपयोिगता पे टट अनु दान से सं बंिधत
ावधान मौजूद नहीं है । यूिटिलटी पे टट सु रा दान करने वाले कुछ दे श म यूएसए, बाजील, चीन, ग ीस, जॉिजया, इटली,
जापान, कोिरया, कुवै त, मले िशया, पे न, पु तगाल, यूएई, पोल ड, पे  आिद ह ।

इसिलए जो भारतीय कंपिनयां और टाट अप जो उपयोिगता पे टट सं रण चाहते ह , केवल उपयु त दे श म ही आवे दन कर
सकते ह ।

https://pib.gov.in/Pressreleaseshare.aspx?PRID=1557418

http://www.iitk.ac.in/siic/d/content/types-patent

58 भारत के अंतद शीय जल संसाधनों के बारे म िननिलिखत पर िवचार कीिजए :


1. भारत के सभी राय म खारे पानी के कुल े फल के सदभ म गु जरात पहले थान पर है ।
2. ट क और तालाब के अं तगत अिधकां श जलीय े  भारत के दिणी राय म है ।

उपरोत म से कौन सा सही है / ह 


A. केवल 1
B. केवल 2
C. 1 और 2 दोन
D. कोई नहीं

Your Answer :
Correct Answer : B

Answer Justification :

जहां तक ​जलाशय का सं बंध है , आं ध दे श, गु जरात, कनाटक, मय दे श, महाराट , उड़ीसा और उर दे श जै से मु ख
राय म जलाशय के अं तगत सवािधक े  आता ह ।

उड़ीसा, उर दे श और असम के राय म 77 ितशत से अिधक े  बील, गोखु र, झील और पिरयत पानी के
अं तगत आता है ।
उड़ीसा का खारे पानी के े फल थम थान है इसके बाद गु जरात, केरल और पिचम बं गाल का थान है ।
अं तद शीय जल सं साधन का कुल े फल इस कार, दे श के पांच राय अथात् उड़ीसा, आं ध दे श, गु जरात,
कनाटक और पिचम बं गाल के साथ असमान प से िवतिरत िकया गया है , जो दे श के आधे से अिधक जल िनकाय के
िलए िजमे दार है ।
ट क और तालाब के अं तगत अिधकां श े  दिणी राय आं ध दे श, कनाटक और तिमलनाडु म िथत ह । ये राय
पिचम बं गाल, राजथान और उर दे श के साथ, दे श म ट क और तालाब के तहत कुल े फल का 62 ितशत
िहसा ह ।

59 सोिडयम ब जोएट का उपयोग िननिलिखत म िकस प म िकया जाता है 


1. भोजन को खराब होने से बचाने के िलए पिररक
2. कृिम वीटनर

नीचे िदए गए कू ट का उपयोग करके सही उर चु िनए :


A. केवल 1
B. केवल 2
C. 1 और 2 दोन

https://telegram.me/UpscPdfDrive
https://telegram.me/UPSC_PDF
Total Marks : 200
Online Prelims Full Length Test - 4
( InsightsIAS Mock Test Series for UPSC Preliminary Exam 2020 ) Mark Scored : 0

D. उपरोत म से कोई नहीं

Your Answer :
Correct Answer : A

Answer Justification :

जे ली, जै म और सोडा जै से अलीय उपाद सोिडयम ब जोएट का उपयोग करते ह ।

यह हाइपरमोनिमया, एक दुलभ िवकार जो रत म अमोिनया को जमा करने का कारण बनता है , के इलाज के िलए एक दवा के
प म भी योग िकया जाता है ।

सोिडयम ब जोएट अिधक माा म सं भािवत प से मानव के िलए हािनकारक हो सकता है ; जै से उपभोताओं के आग ह पर,
कोका कोला कंपनी डाइट कोक से सोिडयम ब जोएट को चरणब तरीके से बाहर करने की िया म है ।

60 मायकोटॉिसन, कुछ सूमजीव ारा िनिमत एक िवषात मे टाबोलाइट है , जो मनु य और अय जानवर म बीमारी और
मृ यु का कारण बनने म सम है । िननिलिखत म से कौन सा सूम जीव मायकोटॉिसन का उपादन करता है 
1. जीवाणु
2. वायरस
3. कवक

नीचे िदए गए कू ट का उपयोग करके सही उर चु िनए :


A. केवल 1 और 2
B. केवल 3
C. केवल 2
D. 1, 2 और 3

Your Answer :
Correct Answer : B

Answer Justification :

मायकोटॉिसन िवषात यौिगक ह जो वाभािवक प से कुछ कार के सांच (कवक) ारा िनिमत होते ह । Molds जो
mycotoxins का उपादन कर सकते ह वे कई खा पदाथों जै से अनाज, सूखे फल, नट् स और मसाल पर िवकिसत होते ह ।
अिधकां श मायकोटॉिसन रासायिनक प से िथर होते ह और खा सं करण से बचे रहते ह ।

माइकोटोिसन के ितकू ल वाय भाव तीव िवषातता से ले कर दीघकािलक भाव जै से िक ितरा की कमी और कसर
तक होते ह ।

मायकोटॉिसन का एसपोजर सीधे सं िमत भोजन खाने से या अय प से दिू षत चारा खाने वाले पशु ओं से हो सकता
है ।

https://www.who.int/news-room/fact-sheets/detail/mycotoxins

61 संसद ारा पािरत ये क िवधेयक कानून बनने से पहले राटपित के पास वीकृित के िलए भे जा जाता है । इसके संदभ म
िननिलिखत पर िवचार कर
1. राट पित िवधे यक को सं सद वापस पु निवचार के िलए भे ज सकता है ।

https://telegram.me/UpscPdfDrive
https://telegram.me/UPSC_PDF
Total Marks : 200
Online Prelims Full Length Test - 4
( InsightsIAS Mock Test Series for UPSC Preliminary Exam 2020 ) Mark Scored : 0

2. राट पित ारा इन िवधे यक को अनु मोिदत करने के िलए सं िवधान म कोई समय सीमा उिलिखत नहीं है ।
3. राट पित, सं सद म िवधायी ताव को भे जने और तु त करने के िलए सं वैधािनक प से अिधकृत है ।

नीचे िदए गए कोड का उपयोग करके सही उर चु न


A. केवल 1 और 2
B. केवल 2
C. केवल 1 और 3
D. 1, 2 और 3

Your Answer :
Correct Answer : A

Answer Justification :

राट पित िवधे यक को सं सद वापस पु निवचार के िलए भे ज सकता है । ऐसा करते हुए, राट पित अपने िववे कािधकार का
उपयोग करता है ।

राट पित के पास वीटो शित होती है िजसके ारा वह सं सद ारा पािरत िबल (धन िवधे यक के अलावा) को वीकृित दे ने से
मना कर सकता है या रोक सकता है ।

यह 'वीटो' शित सीिमत है , यिक यिद सं सद उसी िवधे यक को िफर से पािरत करती है और उसे राट पित को वापस भे जती
है , तो, राट पित को उस िवधे यक को वीकृित दान करनी होगी। हालाँ िक, सं िवधान म उस समय सीमा के बारे म कोई
उले ख नहीं िकया गया है िजसके भीतर राट पित को पु निवचार के िलए िबल वापस भे जना चािहए।

इसका मतलब यह है िक राट पित िबना िकसी समय सीमा के िबल को अपने पास लं िबत रख सकते ह । यह राट पित को बहुत
भावी तरीके से वीटो का उपयोग करने की एक अनौपचािरक शित दे ता है । इसे कभी-कभी 'पॉकेट वीटो' के प म जाना
जाता है ।

सं िवधान म ऐसा कोई ावधान नहीं है । भले ही राट पित सं सद का अिभन अं ग है , ले िकन वह दोन सदन की िवधायी
कायवाही का आरं भ अथवा भाग नहीं ले सकता है ।

62 इितहासकारों ारा िदए गए िननिलिखत म से कौन से तक मगध साम ाय की रणनीितक प से लाभद िथित को िदखाते
है ?
1. मगध म चु र माा म लोहे की खदान सु लभ थीं जो अ व श के िलए सं साधन उपलध कराती थीं।
2. गं गा और उसकी सहायक निदयां साम ाय के िलए सते और सु िवधाजनक सं चार का साधन दान करती थी।

उपरोत म से कौन सा सही है / ह 


A. केवल 1
B. केवल 2
C. 1 और 2 दोन
D. कोई नहीं

Your Answer :
Correct Answer : C

Answer Justification :
मगध म चु र माा म लोहे की खदान सु लभ थीं जो अ व श के िलए सं साधन उपलध कराती थीं। इनसे मगध

https://telegram.me/UpscPdfDrive
https://telegram.me/UPSC_PDF
Total Marks : 200
Online Prelims Full Length Test - 4
( InsightsIAS Mock Test Series for UPSC Preliminary Exam 2020 ) Mark Scored : 0

को एक मजबूत से ना का िनमाण करने और अय साम ाय पर रणनीितक बढ़त हािसल करने म सम बनाया।
गं गा और उसकी सहायक निदयां साम ाय के िलए सते और सु िवधाजनक सं चार का साधन दान करती थी। इससे
मगध से ना तथा यापािरक माल को साम ाय के भीतर और बाहर बड़ी दरू ी पर सते दाम पर ले जाया जा सकता था।
इसके अलावा, मगध े  म कृिष उपादक भूिम थी।
हाथी, जो से ना का एक महवपूण घटक थे , मगध के जं गल म पाए जाते थे ।

इन सभी ने मगध को भारत के इितहास म एक दुजय साम ाय बना िदया।

63 ‘यु ितकरण थीिसस’(Rationalization Thesis) का ितपादन िननिलिखत म से िकसके ारा िदया गया था

A. इरिवं ग गोफमै न
B. मै स वे बर
C. चास राइट िमस
D. इमाईल दुखीम

Your Answer :
Correct Answer : B

Answer Justification :

मै स वे बर ारा दी गयी यु ितयु तकरण वह िया थी िजसके मायम से जाद,ू अलौिकक और धािमक िवचार समाज म
सां कृितक महव खो दे ते ह और िवान तथा यावहािरक गणना पर आधािरत िवचार मु ख हो जाते ह ।

उदाहरण के िलए, आधु िनक समाज म िवान ने मौसम के ित हमारी समझ को तक सं गत बनाया है । िवान हवा के गित
और िदशा, हवा और पानी के तापमान, आदता, आिद जै से भौितक तव के बीच बातचीत के पिरणामवप मौसम के पै टन की
याया करता है ।

64 उप-राटपित के चुनाव से संबंिधत सभी संदेहों तथा िववादों की जांच ‘X’ के ारा की जाएगी और उसका िनणय अंितम होगा।
यहाँ पर ‘X’ है :

A. भारत का सवोच यायालय


B. भारत के राट पित
C. भारत का चु नाव आयोग
D. उपरोत म से कोई नहीं

Your Answer :
Correct Answer : A

Answer Justification :

सं िवधान का अनु छे द 71

राट पित या उपराट पित के चु नाव से सं बंिधत या सं बंिधत मामले

1. राट पित या उपराट पित के िनवाचन से उपन या सं सत सभी शं काओं और िववाद की जांच और िविनचय
उचतम यायालय ारा िकया जाएगा और उसका िविनचय अं ितम होगा।

https://telegram.me/UpscPdfDrive
https://telegram.me/UPSC_PDF
Total Marks : 200
Online Prelims Full Length Test - 4
( InsightsIAS Mock Test Series for UPSC Preliminary Exam 2020 ) Mark Scored : 0

2. यिद उचतम यायालय ारा िकसी यित के राट पित या उपराट पित के प म िनवाचन को शूय घोिषत कर िदया
जाता है तो उसके ारा, यथािथित, राट पित या उपराट पित के पद की शितय के योग और कतय के पालन म
उचतम यायालय के िविनचय की तारीख को या उससे पहले िकए गए काय उस घोषणा के कारण अिविधमाय नहीं
हगे ।
3. इस सं िवधान के उपबं ध के अधीन रहते हुए, राट पित या उपराट पित के िनवाचन से सं बंिधत या सं सत िकसी िवषय
का िविनयमन सं सद िविध ारा कर सकेगी।
4. राट पित या उपराट पित के प म िकसी यित के िनवाचन को उसे िनवािचत करने वाले िनवाचकगण के सदय म
िकसी भी कारण से िवमान िकसी िरित के आधार पर नगत नहीं िकया जाएगा।

65 कांगेस सोशिलट पाटी (CSP) के बारे म िननिलिखत पर िवचार कर


1. वातं य
् ोर यु ग म CSP का गठन भारतीय राट ीय कां गेस (INC) के भीतर हुआ था।
2. समाजवािदय ने भारतीय राट ीय कां गेस (INC) की कुछ पूंजीवादी नीितय की आलोचना की और लोकतां िक समाजवाद
की िवचारधारा म िववास िकया।

नीचे िदए गए कोड का उपयोग करके सही उर चु न


A. केवल 1
B. केवल 2
C. 1 और 2 दोन
D. उपरोत म से कोई नहीं

Your Answer :
Correct Answer : B

Answer Justification :

समाजवादी पाटी की उपि वतं ता पूव यु ग म भारतीय राट ीय कां गेस के जन आदोलन के दौरान से हुई।

कां गेस सोशिलट पाटी (CSP) का गठन कां गेस के भीतर 1934 म यु वा ने ताओं के एक समूह ारा िकया गया थे । 1948 म ,
अपने सदय को दोहरी पाटी की सदयता से रोकने के िलए कां गेस ने अपने सं िवधान म सं शोधन िकया। इसने समाजवािदय
को 1948 म एक अलग सोशिलट पाटी बनाने के िलए मजबूर िकया।

समाजवािदय को लोकतां िक समाजवाद की िवचारधारा पर िववास था िजसने उह कां गेस के साथ-साथ कयु िनट से
अलग िकया। उहने पूँजीपितय और जमींदार के प म और मज़दरू  और िकसान की उपे ा के िलए कां गेस की आलोचना
की।

ले िकन समाजवािदय को उस समय दुिवधा का सामना करना पड़ा जब 1955 म कां गेस ने समाज के समाजवादी वप को
अपना लय घोिषत िकया। इस कार समाजवािदय के िलए खु द को कां गेस के एक भावी िवकप के प म तु त करना
मु िकल हो गया। राममनोहर लोिहया के ने तृव म उनम से कुछ ने कां गेस पाटी से अपनी दरू ी बढ़ाई और आलोचना की।

अशोक मे हता जै से कुछ अय लोग ने कां गेस के साथ सीिमत सहयोग की वकालत की।

जयकाश नारायण, अयु त पटवधन, अशोक मे हता, आचाय नर द दे व, राममनोहर लोिहया और एस.एम. जोशी समाजवादी
दल के ने ताओं म से थे ।

https://telegram.me/UpscPdfDrive
https://telegram.me/UPSC_PDF
Total Marks : 200
Online Prelims Full Length Test - 4
( InsightsIAS Mock Test Series for UPSC Preliminary Exam 2020 ) Mark Scored : 0

66 िननिलिखत मुख अंतर-राय जल िववादों पर िवचार कर


1. गोदावरी जल िववाद यायािधकरण : महाराट और आं ध दे श व अय
2. कावे री िववाद : कनाटक और पु दुचेरी व अय
3. रावी तथा यास िववाद : राजथान और पं जाब व अय

उपरोत म से कौन सा यु म सही सु मेिलत ह 


A. केवल 1 और 2
B. केवल 2 और 3
C. केवल 1 और 3
D. 1, 2 और 3

Your Answer :
Correct Answer : A

Answer Justification :

मु ख अं तर-राय जल िववाद यायािधकरण तथा सं बंिधत प

1. गोदावरी जल िववाद यायािधकरण : महाराट , आं ध दे श, कनाटक, मय दे श और ओिडशा


2. कृणा जल िववाद यायािधकरण –I: महाराट , आं ध दे श, कनाटक
3. नमदा जल िववाद यायािधकरण : राजथान, मय दे श, गु जरात और महाराट
4. रिव एं ड यास वाटर िट यूनल : पं जाब, हिरयाणा और राजथान
5. कावे री जल िववाद यायािधकरण : केरल, कनाटक, तिमलनाडु और पु ड्डुचे री
6. कृणा जल िववाद यायािधकरण –II: कनाटक, ते लंगाना, आं ध दे श और महाराट

67 धानमंी सुरित मातृ व अिभयान (PMSMA) के संदभ म िननिलिखत पर िवचार कर


1. इसका उे य सरकारी वाय कद और अपताल म गभवती मिहलाओं को मु त वाय जांच सु िवधा दान कराना है ।
2. यह सरकार ारा सं चािलत िकसी भी कद म पं जीकृत तनपान कराने वाली मां को एकमु त िवीय अनु दान ारा सं थागत
सव को ोसािहत करता है ।
3. यह िजला मु यालय ारा बं िधत इले ट ॉिनक णाली के मायम से माँ और नवजात बचे के टीकाकरण िरकॉड को टै क
करता है ।

नीचे िदए गए कोड का उपयोग करके सही उर चु न


A. केवल 1
B. केवल 2 और 3
C. केवल 2
D. 1 और 2

Your Answer :
Correct Answer : A

Answer Justification :

धानमं ी सु रित मातृ व अिभयान (PMSMA) कायम का उे य गभवती मिहलाओं को िन : शु क सवोर से वाएं
(एएनसी) दान करना और ये क माह की 9 तारीख को मु त म आवयक उपचार करना है ।

https://telegram.me/UpscPdfDrive
https://telegram.me/UPSC_PDF
Total Marks : 200
Online Prelims Full Length Test - 4
( InsightsIAS Mock Test Series for UPSC Preliminary Exam 2020 ) Mark Scored : 0

योजना के उे य ह

1. गभवती मिहलाओं को वथ जीवन दान करना।


2. मातृ व मृ यु दर को कम करना।
3. गभवती मिहलाओं को उनके वाय सं बंधी मु  और बीमािरय के बारे म जागक करना।
4. िशशु का सु रित सव और वथ जीवन सु िनिचत करना।
यह योजना केवल 3 से 6 माह की गभवती मिहलाओं के िलए है । यह गभवती मिहलाओं को सभी कार के मे िडकल
चे कअप पूरी तरह से मु त दान करे गा।
ये चे कअप दे शभर के मे िडकल स टर, सरकारी और िनजी अपताल और िनजी लीिनक म होगा।

माँ और नवजात बचे के टीकाकरण आं िशक प से मातृ -िशशु टै िकंग णाली ारा पूरी की जाती है । यह एक अलग योजना है
और पहले सीजी ारा शािसत की जा रही है ।

68 इसने ‘सहायक संिध’ णाली को लागू िकया तथा ‘भारत म िब िटश साम ाय’, ‘भारत का िब िटश साम ाय’ बनाने के िलए
‘फॉरवड नीित’ को शु  िकया। वह था :

A. िरचड कोली वे लेजली


B. रॉबट बु लवर िलटन
C. जोसे फ कोइस डुले स
D. ेडिरक है िमटन डफिरन

Your Answer :
Correct Answer : A

Answer Justification :

गवनर जनरल के प म िरचड कोली वे लेजली की िनयु ित िबिटश भारत के इितहास म एक यु ग का तीक है ।

वे लेजली भारत म ‘भारत म िबिटश साम ाय’, ‘भारत का िबिटश साम ाय’ बनाने के िलए ‘फॉरवड नीित’ शु  करने के सं कप
के साथ भारत आए थे । अपने उे य को ात करने के िलए उसने जो णाली अपनाई वह ‘सहायक सं िध’ के नाम से जानी
जाती है ।

69 मासवादी िवचारधारा म , ‘वग संघष ’ का िननिलिखत म से या पिरणाम हो सकता है ?


1. सता म
2. भु खमरी
3. सं साधन के िलए यु 

उपरोत म से कौन सा सही है / ह 


A. केवल 1
B. केवल 2 और 3
C. 1, 2 और 3
D. केवल 1 और 3

Your Answer :
Correct Answer : C

https://telegram.me/UpscPdfDrive
https://telegram.me/UPSC_PDF
Total Marks : 200
Online Prelims Full Length Test - 4
( InsightsIAS Mock Test Series for UPSC Preliminary Exam 2020 ) Mark Scored : 0

Answer Justification :

वग सं घष वह तनाव या िवरोधाभास है जो समाज म िविभन वगों के लोग के बीच ितपधा सामािजक आिथक िहत और
आवयकत के कारण मौजूद है ।

अिधकां श लोग के िलए वग सं घष कट् टरपं थी सामािजक पिरवतन के िलए काल मास और अराजकतावादी िमखाइल
बाकुिनन के काय मु य प से कद म होते है ।

वग सं घष कई अलग-अलग प ले सकता है : य िहं सा, जै से िक सं साधन और सते म के िलए लड़ी गई यु ; िहं सा,
जै से गरीबी व भु खमरी से मौत आिद।

पूंजीवािदय के िखलाफ मजदरू सं घ का सं घष, वग सं घष का एक महवपूण उदाहरण है । यह वग सं घष मु य प से पूंजीपित
और सवहारा वग के बीच होता है , और काम के घं टे, मजदरू ी का मूय, मु नाफे का िवभाजन, उपभोता वतु ओं की लागत,
सं सद या नौकरशाही पर िनयं ण, और आिथक असमानता पर सं घष का प ले ता है ।

70 िननिलिखत कथनों पर िवचार कर


1. मु दा आपूित और आरित धन के बीच के अनु पात को धन गु णक कहा जाता है ।
2. यिद धन गु णक बढ़ता है , तो ब क अिधक मां ग जमा करने म सम हो सकते ह , िजससे धन की अिधक आपूित हो सकती है ।

उपरोत म से कौन सा सही है / ह 


A. केवल 1
B. केवल 2
C. 1 और 2 दोन
D. कोई नहीं

Your Answer :
Correct Answer : C

Answer Justification :

मु दा आपूित और आरित धन के बीच के अनु पात को धन गु णक कहा जाता है । ब क इस धन को जनता से जमा के प म
ात करते ह और इसे बार-बार ऋण के प म दे ते ह तथा अिधक लाभ कमाते ह ।

यिद धन गु णक म वृ दि् ध होती है , उसी उच-शित वाले धन के साथ, ब क अिधक मां ग जमा करने म सम हगे , और इस
कार उचतर मु दा आपूित सं भव होगी।

71 ‘वै िवक फसल िविवधता टट’ िननिलिखत म िकससे संलन एक कोष है ?

A. अं तराट ीय खा नीित अनु संधान सं थान (IFPRI)


B. खा और कृिष सं गठन
C. िवव यापार सं गठन
D. उपरोत म से कोई नहीं

Your Answer :
Correct Answer : D

https://telegram.me/UpscPdfDrive
https://telegram.me/UPSC_PDF
Total Marks : 200
Online Prelims Full Length Test - 4
( InsightsIAS Mock Test Series for UPSC Preliminary Exam 2020 ) Mark Scored : 0

Answer Justification :

‘वै िवक फसल िविवधता ट ट’ अं तरराट ीय कानून के अं तगत एक थािपत वतं  सं गठन है ।

CGIAR अं तराट ीय कृिष अनु संधान सं घ की ओर से ‘खा और कृिष सं गठन’’ और जै व सु रा इं टरने शनल’ ारा वष
2004 म रोम, इटली म ‘वै िवक फसल िविवधता ट ट’ की थापना की गई थी।
अपने 10 साल के इितहास म , फसल ट ट ने खा सु रा हे तु दुिनया भर म फसल िविवधता के सं रण और
उपलधता को सु िनिचत करने के िलए महवपूण छलां ग लगाई है ।

72 राय लोक से वा आयोग के बारे म िननिलिखत पर िवचार कीिजए :


1. राय लोक से वा आयोग के अय को राट पित ारा िनयु त िकया जाता है ले िकन उसे राट पित और रायपाल दोन ारा
हटाया जा सकता है ।
2. आयोग के अय को हटाने के सं बंध म सवोच यायालय ारा दी गई सलाह सं बंिधत ािधकारी के िलए बायकारी है ।

उपरोत म से कौन सा सही है / ह 


A. केवल 1
B. केवल 2
C. 1 और 2 दोन
D. कोई नहीं

Your Answer :
Correct Answer : B

Answer Justification :

राय लोक से वा आयोग के अय तथा अय सदय केवल राट पित ारा ही हटाए जा सकते ह , रायपाल ारा नहीं। यह
SPSC की वतं ता सु िनिचत करने के िलए एक तं  के प म काय करता है ।

दुयवहार के िलए अय SPSC के अय या िकसी अय सदय को भी हटा सकता है । इस मामले म , राट पित, मामले
की जांच के िलए उचतम यायालय को सं दिभत कर गे ।

यिद उचतम यायालय, जांच के बाद, हटाने के कारण सही पाता है तथा इह पदमु त करने की सलाह दे ता है , तो अय या
सदय को हटा जा सकता है । सं िवधान के ावधान के तहत, सवोच यायालय ारा इस सं बंध म दी गई सलाह राट पित के
िलए बायकारी है ।

हालां िक, सवोच यायालय ारा जांच के दौरान, रायपाल सं बंिधत अय या सदय को िनलं िबत कर सकता है ।

73 िननिलिखत महवपूण जनजातीय नृ य तथा सं बंिधत


राय को सु मेिलत कीिजए :
1. होजिगिर (Hojagiri): िपु रा
2. नगेम (Nongkrem): नागाल ड
3. ढोल-चोलोम : अणाचल दे श
4. कादर नृ तम (Kaadar Nritham): केरल
#114911

नीचे िदए गए कू ट का उपयोग करके सही उर चु िनए :


A. केवल 1 और 4

https://telegram.me/UpscPdfDrive
https://telegram.me/UPSC_PDF
Total Marks : 200
Online Prelims Full Length Test - 4
( InsightsIAS Mock Test Series for UPSC Preliminary Exam 2020 ) Mark Scored : 0

B. केवल 2 और 3
C. केवल 1 और 2
D. केवल 2 और 4

Your Answer :
Correct Answer : A

Answer Justification :

होजिगिर िपु रा का एक िस लोक नृ य है । हजगीरी नृ य यु वा लड़िकय ारा िकया जाता है , जो िवशे ष कार के सं तुलन
कौशल और उपकरण की एक ृं खला दिशत करते ह । यह नृ य एक उसव म तु त िकया जाता है , जो अै ल के महीने म
वािषक प से ‘झुम’ की खे ती के िलए एक थान के चयन से पहले , धन की थानीय दे वी से ाथना करने के िलए मनाया जाता
है । नतक अपने िसर पर एक बोतल के साथ िमट् टी के जग पर खड़े होते ह । एक रोशन दीपक बोतल पर सं तुिलत होता है ।
नगेम ’मे घालय की एक महवपूण लोककथा है । पांच िदन तक मनाया जाने वाला ‘का पांबलां ग-नगेम’ खािसय का एक
मु ख धािमक यौहार है । यह नगेम नृ य के नाम से भी िस है । यह खािसय का एक मु ख धािमक यौहार है । यह पव
हर वष अै ल के दसू रे सताह म िशलां ग म मनाया जाता है । ढोल चोलोम, एक डम नृ य, मिणपु र म होली के दौरान िकए
गए नृ य म से एक है । मिणपु र का थां ग-ता नृ य मिणपु र के राजाओं ारा वितत माशल आट िडल से िवकिसत हुआ था।
ढोल चोलोम ढोल की सं गत करने के िलए गायन और नृ य का एक प है और पु ष नतक के एक समूह ारा िकया जाता है ।
कादर नृ तम (Kaadar Nritham) एक कार का आिदवासी नृ य है िजसम केवल मिहलाएँ भाग ले ती ह । यह कोिच े  के
जं गल के कादर जनजाितय का एक आिदम नृ य है । कलाकार एक अधवृ म खु द को यविथत करते ह । वे अपने हाथ म
अपने कपड़ की नोक को कमर के तर तक पकड़ते ह और इसे नृ य की िविभन लय म लहराने लगते ह । यह धीमे कदम म
एक बहुत ही सरल ले िकन सु िचपूण आिदवासी नृ य है ।

74 वसा म घुलनशील िवटािमन, िजह शरीर म वसा की सहायता से अवशोिषत और संगहीत िकया जाता है , सिमिलत ह :
1. िवटािमन सी
2. िवटािमन के
3. िवटािमन ए
4. िवटािमन बी 12

नीचे िदए गए कू ट का उपयोग करके सही उर चु िनए :


A. केवल 1 और 2
B. केवल 4
C. केवल 3 और 4
D. केवल 2 और 3

Your Answer :
Correct Answer : D

Answer Justification :

ितरा, िवकास, सामाय ने  कायों और जै िवक गितिविधय आिद के िलए िवटािमन की आवयकता होती है ।

िवटािमन दो प म आते ह : पानी म घु लनशील और वसा म घु लनशील। पानी म घु लनशील िवटािमन म बी
िवटािमन और िवटािमन सी की एक ृं खला शािमल है ।
आपको पूरे िदन खा पदाथों से पानी म घु लनशील िवटािमन का से वन करने की आवयकता होती है , यिक आपके
शरीर म उह टोर करने का कोई तरीका नहीं है ।

https://telegram.me/UpscPdfDrive
https://telegram.me/UPSC_PDF
Total Marks : 200
Online Prelims Full Length Test - 4
( InsightsIAS Mock Test Series for UPSC Preliminary Exam 2020 ) Mark Scored : 0

वसा म घु लनशील िवटािमन वसा की मदद से अवशोिषत और सं गहीत होते ह । ये िवटािमन, जो ए, डी, ई और के ह ।

75 उर से दिण के म म भारत के िननिलिखत राटीय उान और वयजीव अभयारयों को यविथत कीिजए :
1. दुधवा
2. बां धवगढ़
3. कीबु ल लामजाओ
4. िसमलीपाल

नीचे िदए गए कू ट का उपयोग करके सही उर चु िनए :


A. 1-3-2--4
B. 3-1-4-2
C. 3-1-2-4
D. 1-2-4-3

Your Answer :
Correct Answer : A

Answer Justification :

दु धवा उर दे श के तराई म एक राट ीय उान और दुधवा टाइगर िरजव का एक िहसा है ।

बांधवगढ़ मय दे श म है । यह बायोडायविसटी पाक रॉयल बं गाल टाइगर की बड़ी आबादी के िलए जाना जाता है । अय
जानवर म सफेद बाघ, त दुए और िहरण शािमल ह ।

कीबुल लामजाओ भारत म मिणपु र राय के िबणु पुर िजले म िथत है । यह दुिनया का एकमा तै रता हुआ पाक और लोकतक
झील का अिभन अं ग है ।

िसमलीपाल एक राट ीय उान और भारतीय राय ओिडशा के मयूरभं ज िजले म एक बाघ अभयारय है ।

https://telegram.me/UpscPdfDrive
https://telegram.me/UPSC_PDF
Total Marks : 200
Online Prelims Full Length Test - 4
( InsightsIAS Mock Test Series for UPSC Preliminary Exam 2020 ) Mark Scored : 0

76 भारतीय तटरक के अपतटीय गती पोत ‘ICGS वरद’ को िननिलिखत म िकस से वा के िलए िनयु त िकया गया है 
1. मलका जलडममय म समु दी डकैती पर िनयं ण
2. भारत के िवशे ष आिथक े  की िनगरानी

नीचे िदए गए कू ट का उपयोग करके सही उर चु िनए :


A. केवल 1
B. केवल 2
C. 1 और 2 दोन
D. उपरोत म से कोई नहीं

Your Answer :
Correct Answer : B

Answer Justification :

मलका जलडममय ICG के अिधकार े  से बाहर है । अतः कथन 1 गलत है ।

इं िडयन कोट गाड भारत के समु दी िहत की रा करता है और समु दी कानून को लागू करता है , भारत के े ीय जल और
िवशे ष आिथक े  सिहत यह भारत की समु दी सीमाओं की रा करता है ।

पूवोर े  के तटरक बल के पिरचालन िनयं ण के अं तगत, ओिडशा के पारादीप म ICGS वरद को तै नात िकया गया है ।
भारत के िवशे ष आिथक े  की िनगरानी के िलए ICGS वरद की तै नाती की जाएगी।

https://telegram.me/UpscPdfDrive
https://telegram.me/UPSC_PDF
Total Marks : 200
Online Prelims Full Length Test - 4
( InsightsIAS Mock Test Series for UPSC Preliminary Exam 2020 ) Mark Scored : 0

ICGS वरिदस ने सात अपतटीय गती जहाज की ृं खला म पांचवां , कदीय रा मं ालय के साथ अपने 2015 अनु बंध के
एक भाग के प म लासन और टु बो ारा िनिमत िकया जा रहा है ।

https://www.insightsonindia.com/2020/02/29/insights-daily-current-affairs-pib-summary-29-february
-2020/

77 ‘1000 िंग इिनिशएिटस’ िननिलिखत म से िकस समुदाय के कयाण पर किदत है ?

A. तटीय समु दाय


B. आिदवासी समु दाय
C. माट िसटी समु दाय
D. भारत के उर-पिचमी े  म मथलीय िनवास थान

Your Answer :
Correct Answer : B

Answer Justification :

भारत सरकार ने भु वने वर (ओिडशा) म ‘थानीय व- शासन म अनु सिू चत जनजाित ितिनिधय की मता िनमाण हे तु
कायम’ के अवसर पर ‘1000 िं ग इिनिशएिटस’ और जीआईएस-आधािरत िं ग एटलस से सं बंिधत सूचनाओं के िलये
एक ऑनलाइन पोट ल लॉच िकया गया।

उे य : ग ामीण भारत के किठन एवं दुगम े  म रह रहे जनजातीय लोग के िलये सु रित एवं पयात जलापूित म सु धार
करना है ।

मु ख िवशे षताऐं :

यह पहल बारहमासी झरन के जल का उपयोग करने म सहायता करे गी िजसका उपयोग जनजातीय े  म पानी की कमी को
दरू करने के िलये िकया जाएगा।

इस पहल म पे यजल के िलये पाइप से जलापूित हे तु बु िनयादी ढाँचे का िनमाण करना शािमल है । िजससे िसं चाई और बै कयाड
पोषण उान के िलये जल की यवथा हो सके, पिरणामतः जनजातीय लोग के िलये थायी आजीिवका के अवसर उपन
िकये जा सकगे ।

https://www.insightsonindia.com/2020/02/28/insights-daily-current-affairs-pib-summary-28-february
-2020/

78 िकसान उपादक संगठन (FPO) के बारे म िननिलिखत पर िवचार कीिजए :


1. यह एक अिधसूिचत वािणियक ब क के अधीन एक गै र-ब िकंग िवीय कंपनी (NBFC) है , जो पूरे भारत म कृिष उपज को
िवतिरत करने म मदद करता है ।
2. लघु कृषक कृिष यापार सं घ (SFAC), FPO के सार के िलए सहायता दान कर रहा है ।

नीचे िदए गए कू ट का उपयोग करके सही उर चु िनए :


A. केवल 1
B. केवल 2
C. 1 और 2 दोन

https://telegram.me/UpscPdfDrive
https://telegram.me/UPSC_PDF
Total Marks : 200
Online Prelims Full Length Test - 4
( InsightsIAS Mock Test Series for UPSC Preliminary Exam 2020 ) Mark Scored : 0

D. उपरोत म से कोई नहीं

Your Answer :
Correct Answer : B

Answer Justification :

िकसान उपादक सं गठन’ का अिभाय िकसान, िवशे ष प से छोटे और सीमांत िकसान के समूह से होता है । इस कार के
सं गठन का मु ख उे य कृिष से सं बंिधत चु नौितय के भावी हल की खोज करना होता है । FPO ाथिमक उपादक जै से-
िकसान, दधू उपादक, मछुआर, बु नकर और कारीगर आिद ारा गिठत क़ानूनी इकाई होती है ।

FPO को भारत सरकार तथा नाबाड जै से सं थान से भी सहायता ात होती है । लघु कृषक कृिष यापार सं घ (SFAC), FPO
के सार के िलए सहायता दान कर रहा है ।

PO की आवयक िवशे षताएं

यह उपादक के समूह ारा या तो खे त या गै र-कृिष गितिविधय के िलए बनाया गया है ।


यह एक पं जीकृत िनकाय और कानूनी इकाई है ।
िनमाता सं गठन म शे यरधारक ह ।
यह ाथिमक उपज / उपाद से सं बंिधत यावसाियक गितिविधय से सं बंिधत है ।
यह सदय उपादक के लाभ के िलए काम करता है ।
लाभ का एक िहसा उपादक के बीच साझा िकया जाता है ।
अिधशे ष का बाकी यापार िवतार के िलए अपने वािमव वाले फंड म जोड़ा जाता है ।

https://www.insightsonindia.com/2020/02/28/insights-daily-current-affairs-pib-summary-28-february
-2020/

79 िसिवल से वाओं के संदभ म िननिलिखत कथनों पर िवचार कीिजए :


1. सं सद, माशल लॉ के दौरान सं घ या राय की से वा म िनयु त यितय के ारा यवथा बनाये रखने अथवा सामाय िथित
की बहाली के िलए की गई िकसी भी कारवाई के सं बंध म यितय की रा करने के िलए अिधकृत है ।
2. अिखल भारतीय से वाएं राय सरकार के अं ितम िनयं ण म ह , हालां िक िसिवल से वक को कद ारा िनयु त िकया जाता है और
यूपीएससी ारा अनु शंिसत िकया जाता है ।

उपरोत म से कौन सा सही है / ह 


A. केवल 1
B. केवल 2
C. 1 और 2 दोन
D. कोई नहीं

Your Answer :
Correct Answer : A

Answer Justification :

अनु छे द 33 तथा 34 सं सद को माशल लॉ के दौरान सं घ या राय की से वा म िनयु त यितय के ारा यवथा बनाये रखने
अथवा सामाय िथित की बहाली के िलए की गई िकसी भी कारवाई के सं बंध म यितय की रा करने के िलए अिधकृत
करता है ।

https://telegram.me/UpscPdfDrive
https://telegram.me/UPSC_PDF
Total Marks : 200
Online Prelims Full Length Test - 4
( InsightsIAS Mock Test Series for UPSC Preliminary Exam 2020 ) Mark Scored : 0

इन ावधान के आधार पर सश बल िवशे ष अिधकार अिधिनयम बनाया गया है । इस अिधिनयम ने कुछ अवसर पर लोग
और सश बल के बीच तनाव पै दा िकया है ।

अिखल भारतीय से वाएँ भारत के पूरे े  म काय करती ह और इन से वाओं के िलए चु ने गए अिधकारी राय के शासन म
काय करते ह ।

ले िकन, इन अिधकािरय के िलए, जै से िक IAS का सं बंध कद सरकार के अं ितम िनयं ण म है , राय अनु शासनामक कारवाई
नहीं कर सकते और न ही वे इन अिधकािरय को से वा से हटा सकते ह ।

80 िननिलिखत कथनों पर िवचार कीिजए :


1. िवव जलवायु समे लन, जै व िविवधता अिभसमय (CBD) के पकार का वािषक समे लन (COP) है ।
2. UNFCCC के सभी सदय सं बंिधत पािट य के समे लन (COP) म ितिनिधव करते है , जहां वे अिभसमय के कायावयन
की समीा करते ह ।

नीचे िदए गए कू ट का उपयोग करके सही उर चु िनए :


A. केवल 1
B. केवल 2
C. 1 और 2 दोन
D. उपरोत म से कोई नहीं

Your Answer :
Correct Answer : B

Answer Justification :

िवव जलवायु समे लन, पािट य (COP) का वािषक समे लन और जलवायु पिरवतन पर सं युत राट ेमवक कव शन
(UNFCCC) का सवोच िनकाय है । कव शन के सभी प को COP म ितिनिधव िदया जाता है , जहां वे कव शन के
कायावयन और COP ारा अपनाई गई िकसी भी अय कानूनी उपकरण की समीा करते ह तथा सं युत प से वे
सं थागत और शासिनक यवथा सिहत कव शन के भावी कायावयन को बढ़ावा दे ने के िलए आवयक िनणय ले ते ह ।

2015 म , पे िरस म COP 21 पर, पहले जलवायु पिरवतन समझौते पर सहमित हुई, िजसम सभी 195 सदय राय को
UNFCCC के िलए ितब िकया गया।

https://www.international-climate-initiative.com/en/about-the-iki/iki-at-conferences/world-climate-co
nference

81 कपास (COMPAS) एक अंतरराटीय काय म है , िजसे िडजाइन िकया गया है :

A. उच समु द तथा दुगम थान म पट राते िनिदट करने हे तु
B. राट  को सु रित और एिटे ड ने िवगे शन से वाएं दान करने हे तु
C. थानीय प से ासं िगक अं तजात िवकास के तरीक म सु धार
D. राट  के बीच ई-कचरे के सं करण का बे हतर समवय

Your Answer :
Correct Answer : C

https://telegram.me/UpscPdfDrive
https://telegram.me/UPSC_PDF
Total Marks : 200
Online Prelims Full Length Test - 4
( InsightsIAS Mock Test Series for UPSC Preliminary Exam 2020 ) Mark Scored : 0

Answer Justification :

COMPAS (Comparing and Supporting Endogenous Development) एक अं तरराट ीय कायम है िजसे ग ामीण
लोग की समझ व ान की िविवधता को समझने के िलए बनाया गया है । यह िकसान के वै िवक दृिटकोण के साथ थानीय
योग को ोसािहत करता है एवं िकसान के अनु भव और वदे शी ान पर अं तर-सां कृितक सं वाद करता है ।

अं तजात िवकास थानीय लोग के िवकास के अपने मानदं ड पर आधािरत होता है िजसमे लोग के पास उपलध सामग ी,
सामािजक और आयािमक कयाण को यान म रखता है ।

COMPAS ने टवक की गितिविधयाँ ाकृितक सं साधन बं धन, िशा, वाय और शासन णािलय से सं बंिधत ह । कपास
ने टवक म अीका, एिशया, यूरोप और लै िटन अमे िरका म गै र सरकारी सं गठन और िवविवालय भागीदार ह ।

http://www.compasnet.org/compas_1.html

82 भारत म ‘रोडोड डन’ ाकृितक प से िननिलिखत म िकन दे शों म पाए जाते ह 

A. उराखं ड, िसिकम और पिचम बं गाल


B. पिचम बं गाल, ओिडशा, मय दे श और छीसगढ़
C. केरल, कनाटक और तिमलनाडु
D. गु जरात और महाराट

Your Answer :
Correct Answer : A

Answer Justification :

रोडोड डन पूवी और पिचमी िहमालय दोन म पाया जाता है ।

रोडोड डोन लगभग सभी िहमालयी राय जै से पूवोर भारत, िसिकम, पिचम बं गाल, उराखं ड, जमू-कमीर, िहमाचल
दे श आिद म पाए जा सकते ह ।

यह िसिकम और उराखं ड का राय तीक है ।

83 इनम से कौन सा कथन ‘यायालय की अवमानना’ के संदभ म सही है / ह ?


1. सं िवधान, यायपािलका को अदालत की अवमानना करने का नोिटस जारी करने का अिधकार दे ता है ।
2. उच यायपािलका के िकसी भी यायाधीश के िखलाफ ऐसा कोई नोिटस जारी नहीं िकया जा सकता है ।

उपरोत म से कौन सा सही है / ह 


A. केवल 1
B. केवल 2
C. 1 और 2 दोन
D. कोई नहीं

Your Answer :
Correct Answer : A

https://telegram.me/UpscPdfDrive
https://telegram.me/UPSC_PDF
Total Marks : 200
Online Prelims Full Length Test - 4
( InsightsIAS Mock Test Series for UPSC Preliminary Exam 2020 ) Mark Scored : 0

Answer Justification :

सं िवधान का अनु छे द 129 तथा अनु छे द 142 (2) उचतम यायालय को नोिटस जारी करने और उच यायालय के
यायाधीश सिहत िकसी को भी अदालत की अवमानना ​के िलए दं िडत करने म सम बनाता है ।

वतं  भारत के इितहास म पहली बार, सु ीम कोट ने कलका उच यायालय के यायाधीश सी. एस. कणन को याय
शासन को बािधत करने तथा याियक सं था को बदनाम करने के िलए कायरत और से वािनवृ  यायाधीश के बारे म
अपमानजनक प िलखने के िलए अदालती नोिटस जारी िकया।

84 वायवीय जड़ जमीन के ऊपर होती ह । ये िननिलिखत म से िकन दे शो की वनपितयों म पाए जाती ह 

A. मथलीय दे श
B. तटीय दलदल
C. े यरीज
D. टु ं डा े 

Your Answer :
Correct Answer : B

Answer Justification :

राइजोफोरा जै सी म ग ोव जाित म जड़े , िमट् टी सतह से कुछ दरू ी पर तने से फू टती ह । इन जड़ के वायवीय िहसे म
उपिथत रं ध वायूतक ऊतक के ारा जड़ के भूिमगत िहस म गै स िविनमय म सहायक होते ह ।

अय पौध की तु लना म म ग ोव िवशे ष प से िस और िविशट वायवीय जड़ तथा जै िवक प से खारे े  म पनपने की
मता के कारण िभन होते ह ।

वायवीय जड़ िविवध पौध की जाितय म पाए जाती ह , िजनम एिपफाइड्स जै से ऑिक ड, उणकिटबं धीय तटीय दलदली
पे ड़ जै से म ग ोव, सं साधनयु त बरगद आिद शािमल ह ।

85 िननिलिखत म से कौन से काय लोकसभा ारा िकये जाते है ?


1. कराधान के ताव का अनु मोदन
2. सं िवधान सं शोधन िवधे यक को मं जरू ी दे ना
3. आपातकाल की घोषणा को वीकृित दे ना
4. उच यायपािलका के सदय की िनयु ित

नीचे िदए गए कू ट का उपयोग करके सही उर चु िनए :


A. केवल 1, 3 और 4
B. केवल 1, 2 और 3
C. केवल 2, 3 और 4
D. केवल ४

Your Answer :
Correct Answer : B

Answer Justification :

https://telegram.me/UpscPdfDrive
https://telegram.me/UPSC_PDF
Total Marks : 200
Online Prelims Full Length Test - 4
( InsightsIAS Mock Test Series for UPSC Preliminary Exam 2020 ) Mark Scored : 0

कराधान के ताव का अनु मोदन से सं बंिधत मामले जो धन िवधे यक के अं तगत आते ह । धन िवधे यक केवल लोक सभा म
तु त िकए जा सकते ह , राय सभा म नहीं।

सं िवधान सं शोधन िवधे यक को मं जरू ी दे ना : दोन सदन ऐसा कर सकते ह । इस योजन के िलए एक िवधे यक, यिद िकसी भी
सदन म परात हो जाता है , तो सदन के सं युत बै ठक म तु त नहीं िकया जा सकता है ।

आपातकाल की घोषणा को वीकृित दे ना : दोन सदन इस उद्घोषणा को मं जरू ी दे सकते ह । लोकसभा आपातकाल को र भी
कर सकती है ।

उच यायपािलका के सदय की िनयु ित: SC के मामले म , यह राट पित ारा CJI और HC के कुछ विरठतम
यायाधीश के कॉले िजयम की सलाह पर िकया जाता है । लोक सभा, हालां िक, यायपािलका के महािभयोग म भाग ले ता है ।

86 भारत के उर-पूवी रायों की िननिलिखत राजधािनयों पर िवचार कीिजए :


1. आइजोल
2. इफाल
3. कोिहमा
4. ईटानगर

दिण से उर की ओर इन राजधािनय का सही म या है 


A. 1-2-3-4
B. 2-3-1-4
C. 3-2-1-4
D. 1-3-4-2

Your Answer :
Correct Answer : A

Answer Justification :

https://telegram.me/UpscPdfDrive
https://telegram.me/UPSC_PDF
Total Marks : 200
Online Prelims Full Length Test - 4
( InsightsIAS Mock Test Series for UPSC Preliminary Exam 2020 ) Mark Scored : 0

87 1813 म िब िटश संसद को सौ ंपी गयी "पांचवीं िरपोट " तकालीन गहन संसदीय बहसों का आधार बनी। यह िननिलिखत म
िकससे संबंिधत थी

A. भारत म िवविवालय िशा


B. भारत म ईट इं िडया कंपनी का शासन और गितिविधयाँ
C. भारत म औोिगक कर
D. भारत म राजदोही आदोलन

Your Answer :
Correct Answer : B

Answer Justification :

कंपनी ारा 1760 के दशक के मय म बं गाल म अपना शासन थािपत करने के उपरांत इसकी गितिविधय को इं ल ड म
बारीकी से दे खा जाता था तथा उन पर बहस की जाती थी।

कंपनी अिधकािरय के लालच और भ टाचार को े स म यापक प से चािरत िकया जाता था।


िबिटश सं सद ने कंपनी को भारत के शासन पर िनयिमत िरपोट बनाने के िलए मजबूर िकया और कंपनी के मामल म
पूछताछ करने के िलए सिमितय को िनयु त िकया।
‘पांचवीं िरपोट ’ एक ऐसी िरपोट थी जो एक वर सिमित ारा िनिमत की गई थी।
यह िरपोट भारत म ईट इं िडया कंपनी के शासन की कृित पर गहन सं सदीय बहस का आधार बनी।

88 धानमंी के कत यों से संबंिधत िननिलिखत म कौन से संवैधािनक ावधान ह ?


1. सं घ के कायकलाप के शासन सं बंधी और िवधान िवषयक ाथापनाओं सं बंधी मं िपिरषद के सभी िविनचय राट पित के
िलए सं सिू चत करना

https://telegram.me/UpscPdfDrive
https://telegram.me/UPSC_PDF
Total Marks : 200
Online Prelims Full Length Test - 4
( InsightsIAS Mock Test Series for UPSC Preliminary Exam 2020 ) Mark Scored : 0

2. राट पित के मां गने पर, सं घ के कायकलाप के शासन सं बंधी और िवधान िवषयक ाथापनाओं सं बंधी जानकारी दे ना

नीचे िदए गए कू ट का उपयोग करके सही उर चु िनए :


A. केवल 1
B. केवल 2
C. 1 और 2 दोन
D. उपरोत म से कोई नहीं

Your Answer :
Correct Answer : C

Answer Justification :

अनु छे द 78 के अनु सार :

धानमं ी का यह कतय होगा िक वह --

1. सं घ के कायकलाप के शासन सं बंधी और िवधान िवषयक थापनाओं सं बंधी मं ि-पिरषद के सभी िविनचय
राट पित को सं सिू चत करे ;
2. सं घ के कायकलाप के शासन सं बंधी और िवधान िवषयक थापनाओं सं बंधी जो जानकारी राट पित माँ ग , वह दे ;
और
3. िकसी िवषय को िजस पर िकसी मं ी ने िविनचय कर िदया है िकतु मं ि-पिरषद ने िवचार नहीं िकया है , राट पित
ारा अपे ा िकए जाने पर पिरषद के सम िवचार के िलए रखे ।

89 भारत और पािकतान के बीच समझौतों के बारे म िननिलिखत कथनों पर िवचार कीिजए :


1. िशमला समझौते ने 1999 के कारिगल यु  के पिरणाम को उलटने की कोिशश की।
2. लाहौर घोषणा म परमाणु दौड़ से बचने की िदशा म दोन दे श के ने तृव की िजमे दारी तय की गयी।

उपरोत म से कौन सा सही है / ह 


A. केवल 1
B. केवल 2
C. 1 और 2 दोन
D. कोई नहीं

Your Answer :
Correct Answer : B

Answer Justification :

िशमला समझौते ने 1971 के यु  के पिरणाम को उलटने की कोिशश की (अथात सै िनक की वापसी और यु  बं िदय का
आदान-दान करने के िलए)। यह भारत और पािकतान के बीच अछे पड़ोसी सं बंध के िलए एक यापक लू िं ट था।

लाहौर सं िध की शतों के तहत, परमाणु शागार के िवकास और परमाणु हिथयार के आकिमक और अनिधकृत पिरचालन
उपयोग से बचने के िलए एक आपसी समझ बन गई थी। लाहौर घोषणा ने 1998 के सावजिनक प से परमाणु परीण के बाद
ऐितहािसक प से तनावपूण द्िवपीय सं बंध पर काबू पाने म एक बड़ी सफलता का सं केत िदया तथा परमाणु दौड़ से बचने
की िदशा म दोन दे श के ने तृव की िजमे दारी तय की गयी।

https://telegram.me/UpscPdfDrive
https://telegram.me/UPSC_PDF
Total Marks : 200
Online Prelims Full Length Test - 4
( InsightsIAS Mock Test Series for UPSC Preliminary Exam 2020 ) Mark Scored : 0

90 सामायतः भारत का झंडा पूरे भारत म आधे मतूल पर िननिलिखत म िकसकी मृ यु होने पर फहराया जाता है 
1. राट पित
2. उपाय
3. धान मं ी
4. भारत के मु य यायाधीश

नीचे िदए गए कू ट का उपयोग करके सही उर चु िनए :


A. केवल 1 और 2
B. केवल 2, 3 और 4
C. केवल 1, 2 और 3
D. केवल 1, 3 और 4

Your Answer :
Correct Answer : C

Answer Justification :

भारत का झं डा पूरे भारत म आधे मतूल पर केवल राट पित, उप-राट पित और धानमं ी की मृ यु होने पर फहराया जाता
है ।

लोकसभा अय और भारत के मु य यायाधीश के िलए, राट ीय वज को िदली म तथा उनके गृ ह राय म आधे मतूल
पर फहराया जाता है ।

91 िननिलिखत म कौन सा सबसे यापक प से उपयोग िकए जाने वाला पादप िवकास िनयामक है , िजसे पपीता, सपोटा और
आम के फलों को तीवता से पकाने म िकया जाता है 

A. साइटोिकिनन (Cytokinin)
B. इथे फॉन (Ethephon)
C. िगबेिलन(Gibrellin)
D. औिसन (Auxin)

Your Answer :
Correct Answer : B

Answer Justification :

इथे फॉन सबसे यापक प से इते माल िकया जाने वाला पादप िवकास िनयामक है । पौधे ारा चयापचय करने पर यह
एिथलीन म पिरवितत हो जाता है ।

http://agritech.tnau.ac.in/agriculture/agri_pgr_applications.html

https://telegram.me/UpscPdfDrive
https://telegram.me/UPSC_PDF
Total Marks : 200
Online Prelims Full Length Test - 4
( InsightsIAS Mock Test Series for UPSC Preliminary Exam 2020 ) Mark Scored : 0

92 वै िदक परंपरा म िननिलिखत म से िकन नों का उलेख नहीं िमलता है ?

A. पु नवसु (Punarvasu)
B. मूला (Mula)
C. पु य (Pushya)
D. वितक (Swastika)

Your Answer :
Correct Answer : D

Answer Justification :

ऋवे द म , िकसान को कहा गया है िक वे बार-बार भूिम का सु ख उठाते ह और बीज बोने से पहले उपयु त समय की तीा
करते ह ।

ऋिष पारासर ने कहा था िक अिनल यानी वाित, उराषाढ़ा, उराभादपद, उराफागु नी, रोिहणी, मृ गिशरा (मृ ग), मूला,
पु नवसु, पु य, वण और हत जु ताई के िलए अछे न ह ।

ऋवे द म छह ऋतु एँ का उले ख ह ।

ग ीम (मई - जून); वषा (जु लाई - अगत); हे मंत (िसतं बर - अटू बर); शरद (नवं बर - िदसं बर); िशिशर (जनवरी - फरवरी) और
वसं त (माच - अै ल)।

93 SIDBI के ाथिमक यावसाियक काये म शािमल ह :

A. िनयात-आयात इकाइयाँ
B. शहरी िकसान
C. गरीबी रे खा के नीचे (BPL) नागिरक
D. लघु उोग इकाइयाँ

Your Answer :
Correct Answer : D

Answer Justification :

िसडबी उन इकाइय म िनवे श पर यान किदत करता है , िजनमे सं यं और मशीनरी म 10 िमिलयन पये से अिधक का िनवे श
नहीं होता है । इसके अलावा, SIDBI, पिरवहन, वाय, होटल और पयटन े , बु िनयादी ढांचे, आिद े  के िलए, तथा
छोटे आकार के यावसाियक उम थािपत करने वाले पे शेवर और व-िनयोिजत यितय के िलए भी सहायता दे ता है ।
SIDBI, SC / ST या शारीिरक प से िवकलां ग यितय के िलए पु निव सहायता दान करता है ।
http://exim.indiamart.com/ssi-finance/sidbi.html http://www.sidbi.in/

94 िननिलिखत म कौन से रोग जीवाणु -जिनत है / ह


1. है जा
2. वाइन लू
3. िचकन पॉस
4. ड गू

https://telegram.me/UpscPdfDrive
https://telegram.me/UPSC_PDF
Total Marks : 200
Online Prelims Full Length Test - 4
( InsightsIAS Mock Test Series for UPSC Preliminary Exam 2020 ) Mark Scored : 0

नीचे िदए गए कू ट का उपयोग करके सही उर चु िनए :


A. केवल 1 और 4
B. केवल 2 और 3
C. केवल 1
D. केवल 3 और 4

Your Answer :
Correct Answer : C

Answer Justification :

है जा, िजसे एिशयाई महामारी के प म भी जाना जाता है , एक सं ामक आं शोथ है जो वाइिबयो कॉले री नामक जीवाणु के
एं टेरोटॉिसन उतपन करने वाले उपभे द के कारण होता है ।

वाइन लू, एक सं ामक है जो "सूअर इलु एज़ा वाइरस" नाम के सूम जीव की अने को िविशट कार की जाितय म
से िकसी एक कार के धारक से होती है । 2009 म सं चार मायम ने इसको वाइन लूकहा जो एक नई नल A/H1N1
पै डे िमक वायरस से होता है ।

िचकन पॉस, वे रीसे ला जोटर वायरस से फैलने वाली एक सं ामक बीमारी है । यह बहुत ही सं ामक होती है और सं िमत
िनसृ त पदाथों को सांस के साथ अं दर ले जाने से फैलती है ।

ड गू बु खार एक मछर जिनत उणकिटबं धीय बीमारी है जो ड गू वायरस के कारण होती है ।

95 'समावेश' काय म, िजसे नीित आयोग ारा शु  िकया गया है , के उे य है :
1. िवकास ियाओं को उे िरत करने िविभन मु ख ान और अनु संधान सं थान को एक साथ जोड़ना।
2. िवकिसत दे श ारा दिण एिशया के िवकासशील दे श म सहायता आधािरत ियाओं का सं थागतकरण करना।

उपरोत म से कौन सा सही है / ह 


A. केवल 1
B. केवल 2
C. 1 और 2 दोन
D. कोई नहीं

Your Answer :
Correct Answer : A

Answer Justification :

समावे श – पिरवतनकारी नीितगत सु धार के िलए उकृटता को बढ़ावा दे ना

सं कपना

'समावे श' नीित आयोग ारा िवकास ियाओं को उे िरत करने , सं थागत मता िवकास को बढ़ाने और आपसी सं वधन के
िलए समु दाय के साथ समान तर के इं टरफ़ेस को सम बनाने हे तु िविभन मु ख ान और अनु संधान सं थान को एक साथ
जोड़ने के िलए शु  िकया गया एक कायम है ।

उे य

https://telegram.me/UpscPdfDrive
https://telegram.me/UPSC_PDF
Total Marks : 200
Online Prelims Full Length Test - 4
( InsightsIAS Mock Test Series for UPSC Preliminary Exam 2020 ) Mark Scored : 0

समावे श - अथात समावे शन, सभी तक िवशे ष प से सबसे वं िचत समु दाय तक ान का िवतार करने के िलए एक
दृिटकोण का तीक है और नीित और पित को जोड़ते हुए इसम राट ीय िवकास पहल म उकृट सं थान शािमल
ह । इस पहल के उे य ह :
ितिठत ान और अनु संधान सं थान के साथ ने टविकं ग और साझे दारी के मायम से साय आधािरत नीित
अनु संधान के पािरिथितकी तं  के िलए नीित आयोग की सं थागत मता का िवतार और गहरा करना।
थायी और अिधक समावे शी िवकास ात करने के िलए पिरवतनकारी नीित सु धार म अपनी भूिमका को पूरा करने के
िलए ऐसे सं थान और िहतधारक को सम करना।
िवषयगत ाथिमकताओं / िवकास डोमे न के भीतर और बाहर तथा े  के भीतर और बाहर सं थान और िहतधारक
के बीच ान साझा करने और सूचना के आदान-दान को बढ़ावा दे ना
िवकास ियाओं को उे िरत करने और सं थागत मता िवकास को बढ़ाने के िलए ऐसे सं थान / उकृटता कद
को सम करना।
नीित को पित के साथ जोड़ना - आपसी सं वधन के िलए समु दाय के साथ एक समान तर का इं टरफ़ेस दान करके।

http://www.niti.gov.in/content/concept

96 ाथिमक उपादन महासागरों के गहरे भागों म लगभग शूय रहता है , इसके िननिलिखत म कौन से मु य कारण है 

A. महासागरीय धाराएं
B. पानी का उवे लन
C. सौर काश
D. सफाइड्स की अिधकता

Your Answer :
Correct Answer : C

Answer Justification :

महासागर म काश मं डल के नीचे महासागरीय िनतल के जीव-जं तु काफी हद तक ऊपर सूय की रोशनी म रहने वाले काश
सं ले षक ाथिमक उपादक के भोजन पर िनभर करते ह ।

इसका कारण यह है िक फाइटोल कटन (ाथिमक उपादन) की अछी तरह से वृ दि् ध के िलए चु र कश और पोषक तव
की आवयकता होती है ।

नाइिट फाइं ग और एनारोिबक डे नेिट फाइं ग बै टीिरया के साथ-साथ सफर-िफिसं ग और एनारोिबक सफर-कम करने वाले
बै टीिरया सभी सूरज के अलावा ऊजा ोत का उपयोग करते ह ।

97 ‘िसटी िलवे िबिलटी इं डेस’ िननिलिखत म से िकसके ारा तै यार और कािशत की जाती है 

A. सं युत राट (UN) है िबटे ट


B. शहरी िवकास और गरीबी उमूलन मं ालय
C. म यूरो
D. मु य आिथक सलाहकार, िव मं ालय

Your Answer :
Correct Answer : B

https://telegram.me/UpscPdfDrive
https://telegram.me/UPSC_PDF
Total Marks : 200
Online Prelims Full Length Test - 4
( InsightsIAS Mock Test Series for UPSC Preliminary Exam 2020 ) Mark Scored : 0

Answer Justification :

भारत म ‘िसटी िलवे िबिलटी इं डेस’ शहरी िवकास और गरीबी उमूलन मं ालय, भारत सरकार ारा जारी िकया जाता है ।

शहर का मूयांकन शासन से सं बंिधत 15 मु य मापदं ड, िशा, वाय और सु रा और सु रा, आिथक पहलु ओं
और आवास, खु ले थान, भूिम उपयोग, ऊजा और पानी की उपलधता, ठोस अपिशट बं धन, दष ू ण आिद जै से
भौितक बु िनयादी ढांचे से िकया जाता है ।
शहर को Liveability Index के आधार पर र क िकया जाएगा जो कुल 79 पहलु ओं को कवर करे गा।
यह शहर और कब के बीच वथ ितपधा की भावना सु िनिचत करना है तािक शासन और बु िनयादी ढांचे की
उपलधता म सु धार पर उनका यान किदत िकया जा सके।

98 भारत से अं गेज के जाने के बाद, कुछ िरयासत ने भारतीय सं घ म शािमल होने से इनकार कर िदया। इनम से एक िरयासत थी :

A. झांसी
B. जूनागढ़
C. अवध
D. फरीदकोट

Your Answer :
Correct Answer : B

Answer Justification :

िननिलिखत िरयासत ने शु  म भारतीय सं घ म शािमल होने से इनकार कर िदया :

जूनागढ़ : जूनागढ़ के शासक ने राय के लोग की इछा के िखलाफ पािकतान म शािमल होने की इछा यत की। पटे ल
ने भारतीय सै िनक को भे जा और एक जनमत सं गह के बाद जूनागढ़ भारतीय सं घ म शािमल हो गया।

जमू और कमीर : जब पािकतान से ना के अिधकािरय के ने तृव म पठान जनजाितय ने कमीर पर आमण िकया, तो हिर
िसं ह ने भारत की मदद मां गी। ने ह ने बताया िक अं तरराट ीय कानून के तहत भारत, िरयासत के भारत म िवलय के बाद ही
अपने सै िनक को भे ज सकता है ।

है दराबाद : है दराबाद के मामले म िनज़ाम ने भारतीय सं घ म शािमल होने से इनकार कर िदया। बार-बार अपील के बाद, 1948
म भारतीय सै िनक है दराबाद पर अिधकार कर िलया और िनज़ाम ने आमसमपण कर िदया। अं त म , है दराबाद ने भारतीय सं घ
म वे श िकया।

99 राटीय जै वसंसाधन िवकास बोड (NBDB) के सदभ म िननिलिखत पर िवचार कीिजए :


1. NBDB की थापना पयावरण और वन मं ालय के तवावधान म की गई थी।
2. NBDB ‘िटशू कचर राइड लांट्स’ हे तु राट ीय माणन णाली पर किदत है ।
3. NBDB के ऊजा जै विवान कायम का उे य जै व धन तथा जै व ऊजा को यवहाय बनाने के िलए कम लागत वाली
तकनीक को िवकिसत करना है ।

नीचे िदए गए कू ट का उपयोग करके सही उर चु िनए :


A. केवल 2
B. केवल 1 और 3
C. केवल 2 और 3
D. 1, 2 और 3

https://telegram.me/UpscPdfDrive
https://telegram.me/UPSC_PDF
Total Marks : 200
Online Prelims Full Length Test - 4
( InsightsIAS Mock Test Series for UPSC Preliminary Exam 2020 ) Mark Scored : 0

Your Answer :
Correct Answer : C

Answer Justification :

राट ीय जै वसं साधन िवकास बोड (NBDB) 1999 म जै व ौोिगकी िवभाग था।

NBDB के अं तगत कायम एनजी बायोसाइं स ोग ाम ; बायोोपे िटं ग और बायोिरसोस; मता िनमाण कायम ; ऊतक
सं वधन के िलए राट ीय माणन णाली पादप (NCS-TCP) की थापना की गयी।

डीबीटी के जै व-ऊजा कद ने सभी कार के कृिष अवशे ष और ऊजा फसल से से लुलोिसक इथे नॉल के िलए एक आिथक प
से यवहाय और केले बल तकनीक िवकिसत की है ।

http://www.dbtindia.nic.in/wp-content/uploads/NBDB-inputs.pdf

100 भारत की वतंता के दौरान भारतीय राटीय कांगेस (INC) के अय थे :

A. राज द साद
B. वलभभाई पटे ल
C. जे . बी. कृपलानी
D. जे .एन. ने ह

Your Answer :
Correct Answer : C

Answer Justification :

जे . बी. कृपलानी एक गां धीवादी समाजवादी थे । एक समय पर वह गां धी के सबसे उसाही िशय म से एक थे । कृपलानी ने
1920 के दशक के असहयोग आं दोलन से ले कर 1970 के आपातकाल तक के कायम म भाग िलया।

उह नवं बर 1946 म भारतीय राट ीय कां गेस का अय चु ना गया और सा हतांतरण के महवपूण िदन म सं गठन को
आगे बढ़ाया।

https://telegram.me/UpscPdfDrive

You might also like